Законы кирхгофа кратко: Законы Кирхгофа – simulation, animation – eduMedia

Содержание

2 Закон кирхгофа кратко — Мастер Фломастер

Законы Кирхгофа устанавливают соотношения между токами и напряжениями в разветвленных электрических цепях произвольного типа. Законы Кирхгофа имеют особое значение в электротехнике из-за своей универсальности, так как пригодны для решения любых электротехнических задач. Законы Кирхгофа справедливы для линейных и нелинейных цепей при постоянных и переменных напряжениях и токах.

Первый закон Кирхгофа вытекает из закона сохранения заряда. Он состоит в том, что алгебраическая сумма токов, сходящихся в любом узле, равна нулю.

где – число токов, сходящихся в данном узле. Например, для узла электрической цепи (рис. 1) уравнение по первому закону Кирхгофа можно записать в виде I1 — I2 + I3 — I4 + I5 = 0

В этом уравнении токи, направленные к узлу, приняты положительными.

Физически первый закон Кирхгофа – это закон непрерывности электрического тока.

Второй закон Кирхгофа: алгебраическая сумма падений напряжений на отдельных участках замкнутого контура, произвольно выделенного в сложной разветвленной цепи, равна алгебраической сумме ЭДС в этом контуре

где k – число источников ЭДС; m – число ветвей в замкнутом контуре; Ii , Ri – ток и сопротивление i -й ветви.

Так, для замкнутого контура схемы (рис. 2 ) Е1 — Е2 + Е3 = I1R1 — I2R2 + I3R3 — I4R4

Замечание о знаках полученного уравнения:

1) ЭДС положительна, если ее направление совпадает с направлением произвольно выбранного обхода контура;

2) падение напряжения на резисторе положительно, если направление тока в нем совпадает с направлением обхода.

Физически второй закон Кирхгофа характеризует равновесие напряжений в любом контуре цепи.

Расчет разветвленной электрической цепи с помощью законов Кирхгофа

Метод законов Кирхгофа заключается в решении системы уравнений, составленных по первому и второму законам Кирхгофа.

Метод заключается в составлении уравнений по первому и второму законам Кирхгофа для узлов и контуров электрической цепи и решении этих уравнений с целью определения неизвестных токов в ветвях и по ним – напряжений. Поэтому число неизвестных равно числу ветвей b , следовательно, столько же независимых уравнений необходимо составить по первому и второму законам Кирхгофа.

Число уравнений, которые можно составить на основании первого закона, равно числу узлов цепи, причем только ( y – 1) уравнений являются независимыми друг от друга.

Независимость уравнений обеспечивается выбором узлов. Узлы обычно выбирают так, чтобы каждый последующий узел отличался от смежных узлов хотя бы одной ветвью. Остальные уравнения составляются по второму закону Кирхгофа для независимых контуров, т.е. число уравнений b — (y — 1) = b — y +1 .

Контур называется независимым, если он содержит хотя бы одну ветвь, не входящую в другие контуры.

Составим систему уравнений Кирхгофа для электрической цепи (рис. 3 ). Схема содержит четыре узла и шесть ветвей.

Поэтому по первому закону Кирхгофа составим y — 1 = 4 — 1 = 3 уравнения, а по второму b — y + 1 = 6 — 4 + 1 = 3 , также три уравнения.

Произвольно выберем положительные направления токов во всех ветвях (рис. 4 ). Направление обхода контуров выбираем по часовой стрелке.

Составляем необходимое число уравнений по первому и второму законам Кирхгофа

Полученная система уравнений решается относительно токов. Если при расчете ток в ветви получился с минусом, то его направление противоположно принятому направлению.
Потенциальная диаграмма – это графическое изображение второго закона Кирхгофа, которая применяется для проверки правильности расчетов в линейных резистивных цепях. Потенциальная диаграмма строится для контура без источников тока, причем потенциалы точек начала и конца диаграммы должны получиться одинаковыми.

Рассмотрим контур abcda схемы, изображенной на рис. 4. В ветке ab между резистором R1 и ЭДС E1 обозначим дополнительную точку k.

Рис. 4. Контур для построения потенциальной диаграммы

Потенциал любого узла принимаем равным нулю (например, ?а= 0), выбираем обход контура и определяем потенциалы точек контура: ?а = 0, ?к = ?а — I1R1 , ? b = ? к + Е1, ?с = ? b — I2R2 , ? d = ?c — Е2, ? a = ?d + I3R3 = 0

При построении потенциальной диаграммы необходимо учитывать, что сопротивление ЭДС равно нулю (рис. 5 ).

Рис. 5. Потенциальная диаграмма

Законы Кирхгофа в комплексной форме

Для цепей синусоидального тока законы Кирхгофа формулируются так же, как и для цепей постоянного тока, но только для комплексных значений токов и напряжений.

Первый закон Кирхгофа : «алгебраическая сумма комплексов тока в узле электрической цепи равна нулю»

Второй закон Кирхгофа : «в любом замкнутом контуре электрической цепи алгебраическая сумма комплексных ЭДС равна алгебраической сумме комплексных напряжений на всех пассивных элементах этого контура».

Законы Кирхгофаправила, которые показывают, как соотносятся токи и напряжения в электрических цепях. Эти правила были сформулированы Густавом Кирхгофом в 1845 году. В литературе часто называют законами Кирхгофа, но это не верно, так как они не являются законами природы, а были выведены из третьего уравнения Максвелла при неизменном магнитном поле. Но все же, первое более привычное для них название, поэтому и мы будет их называть, как это принято в литературе – законы Кирхгофа.

Первый закон Кирхгофа – сумма токов сходящихся в узле равна нулю.

Давайте разбираться. Узел это точка, соединяющая ветви. Ветвью называется участок цепи между узлами. На рисунке видно, что ток i входит в узел, а из узла выходят токи i 1 и i 2 . Составляем выражение по первому закона Кирхгофа, учитывая, что токи, входящие в узел имеют знак плюс, а токи, исходящие из узла имеют знак минус i-i 1 -i 2 =0. Ток i как бы растекается на два тока поменьше и равен сумме токов i 1 и i 2 i=i 1 +i 2 . Но если бы, например, ток i 2 входил в узел, тогда бы ток I определялся как i=i 1 -i 2 . Важно учитывать знаки при составлении уравнения.

Первый закон Кирхгофа это следствие закона сохранения электричества: заряд, приходящий к узлу за некоторый промежуток времени, равен заряду, уходящему за этот же интервал времени от узла, т.е. электрический заряд в узле не накапливается и не исчезает.

Второй закон Кирхгофаалгебраическая сумма ЭДС, действующая в замкнутом контуре, равна алгебраической сумме падений напряжения в этом контуре.

Напряжение выражено как произведение тока на сопротивление (по закону Ома).

В этом законе тоже существуют свои правила по применению. Для начала нужно задать стрелкой направление обхода контура. Затем просуммировать ЭДС и напряжения соответственно, беря со знаком плюс, если величина совпадает с направлением обхода и минус, если не совпадает. Составим уравнение по второму закону Кирхгофа, для нашей схемы. Смотрим на нашу стрелку, E2 и Е3 совпадают с ней по направлению, значит знак плюс, а Е1 направлено в противоположную сторону, значит знак минус. Теперь смотрим на напряжения, ток I1 совпадает по направлению со стрелкой, а токи I2 и I3 направлены противоположно. Следовательно:

На основании законов Кирхгофа составлены методы анализа цепей переменного синусоидального тока. Метод контурных токов – метод основанный на применении второго закона Кирхгофа и метод узловых потенциалов основанный на применении первого закона Кирхгофа.

Решение задач на расчет сложных цепей основывается на применении первого и второго законов Кирхгофа, которые наряду с законом Ома являются основными законами электрической цепи.

Законы Кирхгофа определяют распределение токов и напряжений в электрических цепях любой конфигурации.

Рассматривая разветвленные электрические цепи, состоящие из нескольких контуров, нам необходимо установить соотношения между токами, приходящими к любому узлу, и токами, уходящими от него. Из физической сущности электрического тока следует, что общее количество носителей тока, притекающее к узлу в течении некоторого промежутка времени, равно количеству носителей, утекающему от узла за тоже время. Если предположить, что это положение не выполняется, то в узловой точке должно происходить накопление зарядов или убыль — утечка зарядов.

На практике эти явления не наблюдаются, следовательно, мы можем утверждать, что сумма величин токов, притекающих к точке разветвления, равна сумме величин токов, утекающих от нее.

Это положение и является формулировкой первого закона Кирхгофа.

Математическое выражение первого закона Кирхгофа применительно к узлу А:

Условимся токи, притекающие к точке разветвления, считать положительными, а токи, утекающие от нее, — отрицательными и сформулируем окончательно первый закон Кирхгофа:

Алгебраическая сумма величин токов в точке разветвления равна нулю.

Пример

На рисунке изображена узловая точка и указаны направления и величины в пяти ветвях.

Требуется определить величину и направление тока в шестой ветви.

Решение.

Предположим, что ток в шестой ветви притекает к точке А. Используя первый закон Кирхгофа, составим уравнение ∑I=0

Используя первый закон Кирхгофа, можно составить (k-1) уравнений, связывающих между собой величины токов в ветвях. Таким образом, число уравнений на одно меньше, чем число всех узлов цепи. Это объясняется тем, что все токи, входящие в уравнение для узла k, уже вошли в предыдущие уравнения. На схеме в узле А сходятся токи I1, I2, I3; в узле В —I2, I3, I4, I5; в узле С — I4, I5, I1.

Уравнения первого закона Кирхгофа для узлов А и В являются независимыми. В то же время уравнение для узла С. Дает нам зависимость, которая может быть получена на основании уравнений, составленных для первых двух узлов.
В самом деле, на основании первого закона Кирхгофа получим:

Но последнее уравнение не является независимым, так как может быть получено на основании двух первых.
Действительно, складывая (1) и (2), получим

а умножив обе части равенства на -1, будем иметь

Определим теперь число уравнений, которое можно составить, используя второй закон Кирхгофа. Для того чтобы эти уравнения были независимы друг от друга, достаточно чтобы контуры, для которых они пишутся, отличались хотя бы одной ветвью, входящей в их состав.
Математически доказано, что число независимых уравнений m, которое можно составить для любой сложной цепи по второму закону Кирхгофа будет равно

m = n-k + 1 ,

где m —число независимых уравнений, составленных по второму закону Кирхгофа;
n — число ветвей;
к — число узлов.
При выборе контуров стараются по возможности подобрать такие, которые содержат меньшее число ветвей и э. д. с.
Общее число уравнений, составляемых по первому и второму законам Кирхгофа для сложной цепи, состоящей из ветвей и узлов, будет равно числу ветвей.
Складывая число уравнений, составленных на основании первого закона Кирхгофа (k—1), с числом уравнений, составленных на основании второго закона Кирхгофа (m), получим

k — 1 + m = k— 1 + n — k + 1 = n .

Итак, если задана цепь из n ветвей и известны все э. д. с. и сопротивления, всегда можно составить n уравнений по числу неизвестных токов в ветвях.
Для решения задачи на расчет сложной цепи необходимо:

4. Для выбранных узловых точек схемы составить (k — 1) уравнений по первому закону Кирхгофа:

Суммирование токов производится обязательно с учетом знака.
5. Для выбранных замкнутых контуров составить m уравнений по второму закону Кирхгофа:

При составлении этих уравнений э. д. с. суммируются с учетом знака, а падения напряжения берутся со знаком плюс, если направление тока совпадает с направлением обхода контура, и наоборот.
6. Решить систему полученных уравнений, в результате чего определяются величины токов во всех ветвях цепи. Если при решении та или иная величина тока получается со знаком минус, то это значит, что фактическое направление тока в данной ветви обратно тому, которое было принято предварительно.
Для закрепления рассматриваемого порядка расчета сложной цепи с использованием законов Кирхгофа решим пример.

I

Пример. Дана сложная цепь, изображенная на рисунке. Зная Е1, Е2, Е3, r1 r2 и r3, необходимо определить токи в ветвях I1, I2 и I3.

Решение.
1. Анализируя данную схему, устанавливаем, что в ней число ветвей n равно трем, а число узлов k равно двум.
2. Обозначим направление токов в ветвях. Это не значит, что они будут именно такими, как мы предположили. Истинное направление токов определится в ходе решения задачи.
3. Уравнения первого закона Кирхгофа необходимо составить для
(k-1) узлов, или 2-1= 1.
Количество уравнений второго закона Кирхгофа, которое надо составить для решения задачи будет равно

m = n-(k- 1) = 3 — (2 — 1) = 3 — 1=2 .

4. Составим одно уравнение по первому закону Кирхгофа для узла А:

5. Приняв направление обхода контуров против часовой стрелки, составим m-2 уравнений для замкнутых контуров по второму закону Кирхгофа:
— для контура № 1:

6. Решаем систему из трех уравнений.
Из уравнения, составленного по первому закону Кирхгофа (4),
имеем
I1=I2-I3

Подставим полученное значение тока в уравнение (5)

Подставим числовые значения и уравнения (5) и (6).

Упростим эти уравнения и решим их методом подстановки:

Умножим уравнение (7) на 2 и вычтем из полученного результата уравнение (8)

далее, подставляя значение I2 в уравнение (8), получим

5= -3*2,7-4I3; 4I3= -13,1 ;
I3= -13,1/4=-3,3A .

Теперь из уравнения (6) находим ток I1:

В результате решения токи I2 и I1 имеют положительное, а ток I3
отрицательное значение, следовательно, фактическое направление токов I2
и I1 совпадает с принятым, а тока I3— обратно принятому в начале решения задачи.

Закон кирхгофа для электрической цепи для чайников

По каждому проводнику, составляющему электрическую цепь, течет ток. В точке, где проводники сходятся, называемой узлом, справедливо правило: ток суммарный, подтекающий к нему, равняется сумме, оттекающих.

Законы кирхгофа

Другими словами – сколько зарядов подтечет к этой точке за единицу времени, столько же оттечет. Если принять, что приходящий будет «+», а оттекающий – «-», то суммарная его величина будет нулевой.

Это и есть Первый закон кирхгофа для электрической цепи. Смысл его в том состоит, что заряд не накапливается.

Закон Второй, применим к цепи электрической разветвленной.

Эти универсальные законы Кирхгофа применяют очень широко, поскольку позволяют решить множество задач. Большим их достоинство считают простую и понятную всем формулировку, несложные вычисления.

История

Пополнил ряды немецких ученых Кирхгоф в девятнадцатом столетии, когда в стране, находившаяся на пороге революции индустриальной, требовались новейших технологии. Ученые занимались поиском решений, которые могли бы ускорить развитие промышленности.

Активно занимались исследованиями в области электричества, поскольку понимали, что в будущем оно будет широко использоваться. Проблема состояла на тот момент не в том, как составлять электрические цепи из возможных элементов, а в проведении математических вычислений. Тут и появились законы, сформулированные физиком. Они очень помогли.

Алгебраическая сумма приходящих к узлам токов и исходящих из него равна нулю. Эта одновременно вытекает из другого закона — постоянства энергии.

К узлу подходят 2 провода, а отходит один. Значение тока, текущего от узла, такое же, как сумма его, протекающего по двум остальным проводникам, т.е. идущим к нему. Правило Кирхгофа объясняет, что, при ином раскладе, накапливался бы заряд, но такого не бывает. Все знают, что всякую сложную цепь легко разделить на отдельные участки.

Но, при этом непросто определить путь, по которому он проходит. Тем более, что на различных участках сопротивления не одинаковы, поэтому и распределение энергии не будет равномерным.

В соответствие со Вторым правилом Кирхгофа, энергия электронов на каждом из замкнутых участков электрической цепи равняется нулю – нулю равняется всегда в таком контуре суммарное значение напряжений. Если бы нарушилось данное правило, энергия электронов при прохождении определенных участков, уменьшалась бы или увеличивалась. Но, этого не наблюдается.

Применение

Таким образом, благодаря этим двум, выдвинутым Кирхгофом утверждениям, установлено зависимость токов от напряжений в разветвленных участках.

Формула Первого закона такова:

Для схемы, приведенной ниже, справедливо:


I1 — I2 + I3 — I4 + I5 = 0

Плюсовые — это токи, идущие к точке, а те, что выходят из нее «-».

Записывается это так:

  • k — количество ЭДС источников;
  • m – ветви замкнутого контура;
  • Ii,Ri – их сопротивление i-й и ток.

В данной схеме: Е1 — Е2 + Е3 = I1R1 — I2R2 + I3R3 — I4R4.

  • ЭДС принимается «+» при совпадении ее направления с выбранным направлением обхода.
  • При совпадении направления тока и обхода на резисторе, с плюсом будет также напряжение.

Расчет цепи

Способ заключается в умении составления систем уравнений, а также решении их, для нахождения токов в каждой ветви (b), а уже, зная их, умении нахождения величины напряжений.

Проще говоря, количество ветвей совпадать должно с неизвестными величинами в системе. Вначале записывают их, исходя из первого правила: число их идентично с количеством узлов.

Но, независимыми будут (y – 1) выражений. Обеспечивается это выбором, а происходит он так, чтобы разнились они (последующий со смежными) минимум одной ветвью.

Далее, составляются уравнения с использованием второго закона: b — (y — 1) = b — y +1.

Независимым считают контур, содержащий одну (или больше) ветвь, которая в другие не входит.

В качестве примера можно рассмотреть такую схему:

Сдержит она:

узлов – 4;

ветвей –6.

По Первому закону записывают три выражения, т.е. y — 1 = 4 – 1=3.

И столько же на основании Второго, поскольку b — y + 1 = 6 — 4 + 1 = 3.

В ветвях выбирают плюсовое направление и путь обхода (у нас — по стрелке часовой).

Получается:

Осталось относительно токов решить получившуюся систему, понимая, что, когда в процессе решения он получается отрицательным, это свидетельствует о том, что направлен он будет в противоположную сторону.

Правило Кирхгофа применительно к синусоидальным токам

Правила для синусоидального, такие же, как для тока постоянного. Правда, учитываются величины напряжений с комплексными токами.

Первое звучит: «в электрической цепи нулю равна сумма алгебраическая комплексных токов в узле».

Второе правило выглядит так: «алгебраическая сумма ЭДС комплексных в контуре замкнутом равняется сумме алгебраической значений комплексных напряжений, имеющихся на пассивных составляющих данного контура.

Видео: Законы Кирхгофа

Законы Кирхгофа, объясняем простыми словами | Энергофиксик

С помощью первого и второго законов Кирхгофа, а также закона Ома можно найти параметры схемы любой сложности. Поэтому знание, а самое главное понимание этих трех законов строго обязательно для всех кто занимается электроникой. В этой статье я постараюсь максимально просто объяснить и на простейших схемах показать, как работают законы Кирхгофа. Итак, давайте начнем.

Первый закон Кирхгофа

Итак, Первый закон Кирхгофа говорит нам о том, что сумма токов в любом узле абсолютно любой электрической цепи равна нулю. Или так же говорит, что алгебраическая сумма втекающих токов равна алгебраической сумме вытекающих из узла токов.

Узлом в сети называется такой участок цепи, в котором соединяются три и более проводника. Ток, входящий в узел, обозначается стрелочкой, имеющей направление к узлу, а вытекающий — стрелочкой, имеющей направление от узла

И теперь на основании первого закона Кирхгофа запишем следующее уравнение:

Эта же формула может быть записана следующим образом:

При этом положительные и отрицательные знаки токам присвоены условно и если вы поменяете их с точностью до наоборот, то ничего принципиально не изменится.

Для того, чтобы наглядно увидеть работу Первого закона Кирхгофа, давайте соберем простейшую схему.

В качестве источника питания вы можете выбрать абсолютно любой элемент, начиная от пальчиковой батарейки и заканчивая блоком питания с возможностью регулировки.

Примечание. Не обязательно использовать резисторы с номиналом, который указан на схеме. Вы можете подобрать абсолютно любые, какие есть у вас в наличии.

Итак, согласно 1 закону Кирхгофа у нас должно быть верно, следующее уравнение:

Либо верно:

Для проведения практических измерений нам нужно в место на схеме где указан амперметр подключить, например, мультиметр.

Как мы видим по показаниям мультиметра закон работает.

Второй закон Кирхгофа

С пониманием второго закона у многих радиолюбителей в самом начале пути возникают трудности. Но если объяснить по-простому, то все более чем просто, сейчас докажу.

Итак, определение второго закона Кирхгофа звучит так:

В любом замкнутом контуре электрической цепи алгебраическая сумма ЭДС равна алгебраической сумме напряжений на всех пассивных элементах цепи.

Согласитесь, звучит не очень понятно, а вот если сказать проще то:

Сумма ЭДС в замкнутом контуре равна сумме падений напряжений и формула, выражающая этот закон, будет иметь такой вид

Или же

Для понимания давайте разберем самую простую схемку с одним пассивным элементом (резистором) и источником питания в виде пальчиковой батарейки.

Так как у нас резистор один, то падение напряжение на его выводах будет равно величине ЭДС элемента питания (батарейки), то есть 1,5 В = 1,5 В.

Если несколько усложнить схему и добавить к резистору еще один с аналогичным сопротивлением, то в этом случае, то напряжение в 1,5 Вольта поделится пополам на резисторах и будет равно 0,75 В.

Так же произойдет деление напряжения, если мы в цепочку включим третий резистор с одинаковым сопротивлением.

Формула обретет следующий вид:

Давайте для понимания соберем эту схему и произведем измерения.

Как видите, согласно второму закону Кирхгофа, небольшое расхождение в показаниях мультиметра спишем на погрешность прибора (китай как никак).

Кроме одного источника питания в цепи их может быть несколько как, например, в этой схеме

В этом случае у нас два источника питания подключены последовательно встречно, в таком варианте к нашим резисторам будет приложена разность ЭДС, то есть формула обретет следующий вид:

Второй закон Кирхгофа функционирует в цепях независимо от того сколько источников ЭДС и нагрузок будет в схеме. Так же нет принципиальной разницы, где они будут располагаться.

Так же первый и второй законы Кирхгофа одинаково применимы как для постоянного, так и для переменного тока.

Статья оказалась полезна или интересна, тогда ставим лайк и спасибо за уделенное внимание!

4. Законы Ома и Кирхгофа

Закон Ома в простейшем случае связывает величину тока через сопротивление с величиной этого сопротивления и приложенного к нему напряжения:

Сила тока на некотором участке электрической цепи прямо пропорциональна напряжению на этом участке и обратно пропорциональна сопротивлению этого участка.

Закон Ома справедлив для любой ветви (или части ветви) электрической цепи, в таких случаях его называют обобщенным законом Ома. Для ветви, не содержащей ЭДС, закон Ома запишется:

Здесь — потенциалы крайних точек ветви, их разность можно заменить напряжением Uab.

Обобщенный закон Ома для ветви, содержащей ЭДС (т.е. для активной ветви):

Пример: Записать закон Ома для активной цепи на рис. 2.

Первый закон Кирхгофа

Алгебраическая сумма токов, сходящихся в любом узле электрической цепи равна нулю. При этом токи, текущие к узлу считаются положительными, а от узла — отрицательными. Другая формулировка: сумма токов, подходящих к узлу, равна сумме токов, отходящих от узла.

Первый закон Кирхгофа по сути является законом баланса токов в узлах цепи.

Второй закон Кирхгофа

В любом замкнутом контуре электрической цепи алгебраическая сумма падений напряжений на элементах, входящих в контур, равна алгебраической сумме ЭДС.

Второй закон Кирхгофа по сути является законом баланса напряжений в контурах электрических цепей.

Для составления уравнения по 2-му закону Кирхгофа выбирается произвольное направление обхода контура. Тогда, если направление тока в цепи совпадает с направлением обхода, то соответствующее слагаемое берется со знаком «+», а если не совпадает, то со знаком «-«. Аналогичное правило расстановки знаков справедливо и для ЭДС.

Пример:

Уравнение по 2-му закону Кирхгофа может быть записано и для контура, имеющего разрыв цепи, однако при этом необходимо в уравнении учитывать напряжение между точками разрыва.

Пример:

Второе правило кирхгофа пример

В сложных электрических цепях, то есть где имеется несколько разнообразных ответвлений и несколько источников ЭДС имеет место и сложное распределение токов. Однако при известных величинах всех ЭДС и сопротивлений резистивных элементов в цепи мы можем вычистить значения этих токов и их направление в любом контуре цепи с помощью

первого и второго закона Кирхгофа. Суть законов Кирхгофа я довольно кратко изложил в своем учебнике по электронике, на страницах сайта http://www.sxemotehnika.ru.

Пример сложной электрической цепи вы можете посмотреть на рисунке 1.

Рисунок 1. Сложная электрическая цепь.

Иногда законы Кирхгофа называют правилами Кирхгофа, особенно в старой литературе.

Итак, для начала напомню все-таки суть первого и второго закона Кирхгофа, а далее рассмотрим примеры расчета токов, напряжений в электрических цепях, с практическими примерами и ответами на вопросы, которые задавались мне в комментариях на сайте.

Первый закон Кирхгофа

Формулировка №1: Сумма всех токов, втекающих в узел, равна сумме всех токов, вытекающих из узла.

Формулировка №2: Алгебраическая сумма всех токов в узле равна нулю.

Поясню первый закон Кирхгофа на примере рисунка 2.

Рисунок 2. Узел электрической цепи.

Здесь ток I1– ток, втекающий в узел , а токи I2 и I3 — токи, вытекающие из узла. Тогда применяя формулировку №1, можно записать:

Что бы подтвердить справедливость формулировки №2, перенесем токи I2 и I 3 в левую часть выражения (1), тем самым получим:

Знаки «минус» в выражении (2) и означают, что токи вытекают из узла.

Знаки для втекающих и вытекающих токов можно брать произвольно, однако в основном всегда втекающие токи берут со знаком «+», а вытекающие со знаком «-» (например как получилось в выражении (2)).

Можно посмотреть отдельный видеоурок по первому закону Кирхофа в разделе ВИДЕОУРОКИ.

Второй закон Кирхгофа.

Формулировка: Алгебраическая сумма ЭДС, действующих в замкнутом контуре, равна алгебраической сумме падений напряжения на всех резистивных элементах в этом контуре.

Здесь термин «алгебраическая сумма» означает, что как величина ЭДС так и величина падения напряжения на элементах может быть как со знаком «+» так и со знаком «-». При этом определить знак можно по следующему алгоритму:

1. Выбираем направление обхода контура (два варианта либо по часовой, либо против).

2. Произвольно выбираем направление токов через элементы цепи.

3. Расставляем знаки для ЭДС и напряжений, падающих на элементах по правилам:

– ЭДС, создающие ток в контуре, направление которого совпадает с направление обхода контура записываются со знаком «+», в противном случае ЭДС записываются со знаком «-».

– напряжения, падающие на элементах цепи записываются со знаком «+», если ток, протекающий через эти элементы совпадает по направлению с обходом контура, в противном случае напряжения записываются со знаком «-».

Например, рассмотрим цепь, представленную на рисунке 3, и запишем выражение согласно второму закону Кирхгофа, обходя контур по часовой стрелке, и выбрав направление токов через резисторы, как показано на рисунке.

Рисунок 3. Электрическая цепь, для пояснения второго закона Кирхгофа.

Предлагаю посмотреть отдельный видеоурок по второму закону Кирхогфа (теория).

Расчеты электрических цепей с помощью законов Кирхгофа.

Теперь давайте рассмотрим вариант сложной цепи, и я вам расскажу, как на практике применять законы Кирхгофа.

Итак, на рисунке 4 имеется сложная цепь с двумя источниками ЭДС величиной E1=12 в и E2=5 в , с внутренним сопротивлением источников r1=r2=0,1 Ом, работающих на общую нагрузку R = 2 Ома. Как же будут распределены токи в этой цепи, и какие они имеют значения, нам предстоит выяснить.

Рисунок 4. Пример расчета сложной электрической цепи.

Теперь согласно первому закону Кирхгофа для узла А составляем такое выражение:

так как I1 и I 2 втекают в узел А , а ток I вытекает из него.

Используя второй закон Кирхгофа, запишем еще два выражения для внешнего контура и внутреннего левого контура, выбрав направление обхода по часовой стрелке.

Для внешнего контура:

Для внутреннего левого контура:

Итак, у нас получилась система их трех уравнений с тремя неизвестными:

Теперь подставим в эту систему известные нам величины напряжений и сопротивлений:

12 = 0,1I1 +2I.

Далее из первого и второго уравнения выразим ток I2

12 = 0,1I1 + 2I.

Следующим шагом приравняем первое и второе уравнение и получим систему из двух уравнений:

12 = 0,1I1 + 2I.

Выражаем из первого уравнения значение I

I = 2I1– 70;

И подставляем его значение во второе уравнение

Решаем полученное уравнение

12 = 0,1I1 + 4I1 – 140.

12 + 140= 4,1I1

Теперь в выражение I = 2I1– 70 подставим значение

I1=37,073 (А) и получим:

I = 2*37,073 – 70 = 4,146 А

Ну, а согласно первому закона Кирхгофа ток I2=I – I1

I2=4,146 – 37,073 = -32,927

Знак «минус» для тока I2 означает, то что мы не правильно выбрали направление тока, то есть в нашем случае ток I 2 вытекает из узла А .

Теперь полученные данные можно проверить на практике или смоделировать данную схему например в программе Multisim.

Скриншот моделирования схемы для проверки законов Кирхгофа вы можете посмотреть на рисунке 5.

Рисунок 5. Сравнение результатов расчета и моделирования работы цепи.

Для закрепления результатата предлагаю посмотреть подготовленное мной видео:

ПОНРАВИЛАСЬ СТАТЬЯ? ПОДЕЛИСЬ С ДРУЗЬЯМИ В СОЦИАЛЬНЫХ СЕТЯХ!

В сложных электрических цепях, то есть где имеется несколько разнообразных ответвлений и несколько источников ЭДС имеет место и сложное распределение токов. Однако при известных величинах всех ЭДС и сопротивлений резистивных элементов в цепи мы можем вычистить значения этих токов и их направление в любом контуре цепи с помощью первого и второго закона Кирхгофа. Суть законов Кирхгофа я довольно кратко изложил в своем учебнике по электронике, на страницах сайта http://www.sxemotehnika.ru.

Пример сложной электрической цепи вы можете посмотреть на рисунке 1.

Рисунок 1. Сложная электрическая цепь.

Иногда законы Кирхгофа называют правилами Кирхгофа, особенно в старой литературе.

Итак, для начала напомню все-таки суть первого и второго закона Кирхгофа, а далее рассмотрим примеры расчета токов, напряжений в электрических цепях, с практическими примерами и ответами на вопросы, которые задавались мне в комментариях на сайте.

Первый закон Кирхгофа

Формулировка №1: Сумма всех токов, втекающих в узел, равна сумме всех токов, вытекающих из узла.

Формулировка №2: Алгебраическая сумма всех токов в узле равна нулю.

Поясню первый закон Кирхгофа на примере рисунка 2.

Рисунок 2. Узел электрической цепи.

Здесь ток I1– ток, втекающий в узел , а токи I2 и I3 — токи, вытекающие из узла. Тогда применяя формулировку №1, можно записать:

Что бы подтвердить справедливость формулировки №2, перенесем токи I2 и I 3 в левую часть выражения (1), тем самым получим:

Знаки «минус» в выражении (2) и означают, что токи вытекают из узла.

Знаки для втекающих и вытекающих токов можно брать произвольно, однако в основном всегда втекающие токи берут со знаком «+», а вытекающие со знаком «-» (например как получилось в выражении (2)).

Можно посмотреть отдельный видеоурок по первому закону Кирхофа в разделе ВИДЕОУРОКИ.

Второй закон Кирхгофа.

Формулировка: Алгебраическая сумма ЭДС, действующих в замкнутом контуре, равна алгебраической сумме падений напряжения на всех резистивных элементах в этом контуре.

Здесь термин «алгебраическая сумма» означает, что как величина ЭДС так и величина падения напряжения на элементах может быть как со знаком «+» так и со знаком «-». При этом определить знак можно по следующему алгоритму:

1. Выбираем направление обхода контура (два варианта либо по часовой, либо против).

2. Произвольно выбираем направление токов через элементы цепи.

3. Расставляем знаки для ЭДС и напряжений, падающих на элементах по правилам:

– ЭДС, создающие ток в контуре, направление которого совпадает с направление обхода контура записываются со знаком «+», в противном случае ЭДС записываются со знаком «-».

– напряжения, падающие на элементах цепи записываются со знаком «+», если ток, протекающий через эти элементы совпадает по направлению с обходом контура, в противном случае напряжения записываются со знаком «-».

Например, рассмотрим цепь, представленную на рисунке 3, и запишем выражение согласно второму закону Кирхгофа, обходя контур по часовой стрелке, и выбрав направление токов через резисторы, как показано на рисунке.

Рисунок 3. Электрическая цепь, для пояснения второго закона Кирхгофа.

Предлагаю посмотреть отдельный видеоурок по второму закону Кирхогфа (теория).

Расчеты электрических цепей с помощью законов Кирхгофа.

Теперь давайте рассмотрим вариант сложной цепи, и я вам расскажу, как на практике применять законы Кирхгофа.

Итак, на рисунке 4 имеется сложная цепь с двумя источниками ЭДС величиной E1=12 в и E2=5 в , с внутренним сопротивлением источников r1=r2=0,1 Ом, работающих на общую нагрузку R = 2 Ома. Как же будут распределены токи в этой цепи, и какие они имеют значения, нам предстоит выяснить.

Рисунок 4. Пример расчета сложной электрической цепи.

Теперь согласно первому закону Кирхгофа для узла А составляем такое выражение:

так как I1 и I 2 втекают в узел А , а ток I вытекает из него.

Используя второй закон Кирхгофа, запишем еще два выражения для внешнего контура и внутреннего левого контура, выбрав направление обхода по часовой стрелке.

Для внешнего контура:

Для внутреннего левого контура:

Итак, у нас получилась система их трех уравнений с тремя неизвестными:

Теперь подставим в эту систему известные нам величины напряжений и сопротивлений:

12 = 0,1I1 +2I.

Далее из первого и второго уравнения выразим ток I2

12 = 0,1I1 + 2I.

Следующим шагом приравняем первое и второе уравнение и получим систему из двух уравнений:

12 = 0,1I1 + 2I.

Выражаем из первого уравнения значение I

I = 2I1– 70;

И подставляем его значение во второе уравнение

Решаем полученное уравнение

12 = 0,1I1 + 4I1 – 140.

12 + 140= 4,1I1

Теперь в выражение I = 2I1– 70 подставим значение

I1=37,073 (А) и получим:

I = 2*37,073 – 70 = 4,146 А

Ну, а согласно первому закона Кирхгофа ток I2=I – I1

I2=4,146 – 37,073 = -32,927

Знак «минус» для тока I2 означает, то что мы не правильно выбрали направление тока, то есть в нашем случае ток I 2 вытекает из узла А .

Теперь полученные данные можно проверить на практике или смоделировать данную схему например в программе Multisim.

Скриншот моделирования схемы для проверки законов Кирхгофа вы можете посмотреть на рисунке 5.

Рисунок 5. Сравнение результатов расчета и моделирования работы цепи.

Для закрепления результатата предлагаю посмотреть подготовленное мной видео:

ПОНРАВИЛАСЬ СТАТЬЯ? ПОДЕЛИСЬ С ДРУЗЬЯМИ В СОЦИАЛЬНЫХ СЕТЯХ!

Два закона Кирхгофа вместе с законом Ома составляют тройку законов, с помощью которых можно определить параметры электрической цепи любой сложности. Законы Кирхгофа мы будем проверять на примерах простейших электрических схем, собрать которые не составит никакого труда. Для этого понадобится несколько резисторов, пара источников питания, в качестве которых подойдут гальванические элементы (батарейки) и мультиметр.

Первый закон Кирхгофа

Первый закон Кирхгофа говорит, что сумма токов в любом узле электрической цепи равна нулю. Существует и другая, аналогичная по смыслу формулировка: сумма значений токов, входящих в узел, равна сумме значений токов, выходящих из узла.

Давайте разберем сказанное более подробно. Узлом называют место соединения трех и более проводников.

Ток, который втекает в узел, обозначается стрелкой, направленной в сторону узла, а выходящий из узла ток – стрелкой, направленной в сторону от узла.

Согласно первому закону Кирхгофа

Условно присвоили знак «+» всем входящим токам, а «-» ‑ все выходящим. Хотя это не принципиально.

1 закон Кирхгофа согласуется с законом сохранения энергии, поскольку электрические заряды не могут накапливаться в узлах, поэтому, поступающие к узлу заряды покидают его.

Убедиться в справедливости 1-го закона Кирхгофа нам поможет простая схема, состоящая из источника питания, напряжением 3 В (две последовательно соединенные батарейки по 1,5 В), три резистора разного номинала: 1 кОм, 2 кОм, 3,2 кОм (можно применять резисторы любых других номиналов). Токи будем измерять мультиметром в местах, обозначенных амперметром.

Если сложить показания трех амперметров с учетом знаков, то, согласно первому закону Кирхгофа, мы должны получить ноль:

Или показания первого амперметра А1 будет равняться сумме показаний второго А2 и третьего А3 амперметров.

Второй закон Кирхгофа

Второй закон Кирхгофа воспринимается начинающими радиолюбителями гораздо сложнее, нежели первый. Однако сейчас вы убедитесь, что он достаточно прост и понятен, если объяснять его нормальными словами, а не заумными терминами.

Упрощенно 2 закон Кирхгофа говорит: сумма ЭДС в замкнутом контуре равна сумме падений напряжений

ΣE = ΣIR

Самый простой случай данного закона разберем на примере батарейки 1,5 В и одного резистора.

Поскольку резистор всего один и одна батарейка, то ЭДС батарейки 1,5 В будет равна падению напряжения на резисторе.

Если мы возьмем два резистора одинакового номинала и подключим к батарейке, то 1,5 В распределятся поровну на резисторах, то есть по 0,75 В.

Если возьмем три резистора снова одинакового номинала, например по 1 кОм, то падение напряжения на них будет по 0,5 В.

Формулой это будет записано следующим образом:

Рассмотрим условно более сложный пример. Добавим в последнюю схему еще один источник питания E2, напряжением 4,5 В.

Обратите внимание, что оба источника соединены последовательно и согласно, то есть плюс одной батарейки соединяется с минусом другой батарейки или наоборот. При таком способе соединения гальванических элементов их электродвижущие силы складываются: E1 + E2 = 1,5 + 4,5 = 6 В, а падение напряжения на каждом сопротивлении составляет по 2 В. Формулой это описывается так:

И последний отличительный вариант, который мы рассмотрим в данной статье, предполагает последовательное встречное соединение гальванических элементов. При таком соединении источников питания из большей ЭДС отнимается значение меньшей ЭДС. Следовательно к резисторам R1…R3 будет приложена разница E1 – E2, то есть 4,5 – 1,5 = 3 В, — по одному вольту на каждый резистор.

Второй закон Кирхгофа работает не зависимо от количества источников питания и нагрузок, а также независимо от места их расположения в контуре схемы. Полезно будет собрать рассмотренные схемы и выполнить соответствующие измерения с помощью мультиметра.

Законы Кирхгофа действуют как для постоянного, так и для переменного тока.

Реферат — Законы Ома. Законы Кирхгофа


Подборка по базе: лекция 3. Закон Кирхгофа_.pdf, Суть теории учебной деятельности Содержание.docx, Дипломы содержание.docx, Краткое содержание лекций. БИР.КАЗ.2020-2021.docx, Понятие, цели, задачи, содержание прогнозирования..docx, 2 кр законы взаимодействия.doc, 5.10. Кормление, содержание щенной суки и уход за ней..docx, Экономическое содержание понятия.docx, Проект 1. Содержание практической управленческой деятельности.do, Организация и содержание ППС в ОО.doc

РЕФЕРАТ

Дисциплина: Физика

Тема: «Законы Ома. Законы Кирхгофа»
Содержание реферата.

1.Краткая биография Ома, история открытия закона – стр.3.
2.Общий вид закона Ома – стр.5.
3.Виды законов Ома – стр.6.
4.Краткая биография Кирхгофа – стр.8.
5.Общий вид закона Кирхгофа – стр.9.
6.Виды законов Кирхгофа – стр.10.
7.Список использованной литературы – стр.15.

1.Краткая биография Ома, история открытия закона.

Георг Симон Ом родился 16 марта 1787 года в Эрлангене, в семье потомственного слесаря. После окончания школы Георг поступил в городскую гимназию. Гимназия Эрлангена курировалась университетом. Занятия в гимназии вели четыре профессора. Георг, закончив гимназию, весной 1805 года приступил к изучению математики, физики и философии на философском факультете Эрлангенского университета.

Проучившись три семестра, он принял приглашение занять место учителя математики в частной школе швейцарского городка Готтштадта.

В 1811 году он возвращается в Эрланген, заканчивает университет и получает степень доктора философии. Сразу же по окончании университета ему была предложена должность приват-доцента кафедры математики этого же университета.

В 1812 году Ом был назначен учителем математики и физики школы в Бамберге. В 1817 году он публикует свою первую печатную работу, посвященную методике преподавания «Наиболее оптимальный вариант преподавания геометрии в подготовительных классах». Ом занялся исследованиями электричества. В основу своего электроизмерительного прибора Ом заложил конструкцию крутильных весов Кулона. Результаты своих исследований Ом оформил в виде статьи под названием «Предварительное сообщение о законе, по которому металлы проводят контактное электричество». Статья была опубликована в 1825 году в «Журнале физики и химии», издаваемом Швейггером. Однако выражение, найденное и опубликованное Омом, оказалось неверным, что стало одной из причин его длительного непризнания. Приняв все меры предосторожности, заранее устранив все предполагаемые источники ошибок, Ом приступил к новым измерениям.

Появляется в свет его знаменитая статья «Определение закона, по которому металлы проводят контактное электричество, вместе с наброском теории вольтаического аппарата и мультипликатора Швейггера», вышедшая в 1826 году в «Журнале физики и химии».

В мае 1827 года «Теоретические исследования электрических цепей» объемом в 245 страниц, в которых содержались теперь уже теоретические рассуждения Ома по электрическим цепям. В этой работе ученый предложил характеризовать электрические свойства проводника его сопротивлением и ввел этот термин в научный обиход. Ом нашел более простую формулу для закона участка электрической цепи, не содержащего ЭДС: «Величина тока в гальванической цепи прямо пропорциональна сумме всех напряжений и обратно пропорциональна сумме приведенных длин. При этом общая приведенная длина определяется как сумма всех отдельных приведенных длин для однородных участков, имеющих различную проводимость и различное поперечное сечение».

В 1829 году появляется его статья «Экспериментальное исследование работы электромагнитного мультипликатора», в которой были заложены основы теории электроизмерительных приборов. Здесь же Ом предложил единицу сопротивления, в качестве которой он выбрал сопротивление медной проволоки длиной 1 фут и поперечным сечением в 1 квадратную линию.

В 1830 году появляется новое исследование Ома «Попытка создания приближенной теории униполярной проводимости».

Только в 1841 году работа Ома была переведена на английский язык, в 1847 году — на итальянский, в 1860 году — на французский.

16 февраля 1833 года, через семь лет после выхода из печати статьи, в которой было опубликовано его открытие, Ому предложили место профессора физики во вновь организованной политехнической школе Нюрнберга. Ученый приступает к исследованиям в области акустики. Результаты своих акустических исследований Ом сформулировал в виде закона, получившего впоследствии название акустического закона Ома.

Раньше всех из зарубежных ученых закон Ома признали русские физики Ленц и Якоби. Они помогли и его международному признанию. При участии русских физиков, 5 мая 1842 года Лондонское Королевское общество наградило Ома золотой медалью и избрало своим членом.

В 1845 году его избирают действительным членом Баварской академии наук. В 1849 году ученого приглашают в Мюнхенский университет на должность экстраординарного профессора. В этом же году он назначается хранителем государственного собрания физико-математических приборов с одновременным чтением лекций по физике и математике. В 1852 году Ом получил должность ординарного профессора. Ом скончался 6 июля 1854 года. В 1881 году на электротехническом съезде в Париже ученые единогласно утвердили название единицы сопротивления — 1 Ом.
2.Общий вид закона Ома.

Закон Ома устанавливает зависимость между силой токаI в проводнике и разностью потенциалов (напряжением) U между двумя фиксированными точками (сечениями) этого проводника:

(1)

Коэффициент пропорциональности R, зависящий от геометрических и электрических свойств проводника и от температуры, называется омическим сопротивлением или просто сопротивлением данного участка проводника. Закон Ома был открыт в 1826 нем. физиком Г. Омом.

В общем случае зависимость междуIи U нелинейна, однако на практике всегда можно в определенном интервале напряжений считать её линейной и применять закон Ома; для металлов и их сплавов этот интервал практически неограничен.

Закон Ома в форме (1) справедлив для участков цепи, не содержащих источников ЭДС. При наличии таких источников (аккумуляторов, термопар, генераторов и т. д.) закон Ома имеет вид:

(2)

где — ЭДС всех источников, включённых в рассматриваемый участок цепи. Для замкнутой цепи закон Ома принимает вид:

(3)

где — полное сопротивление цепи, равное сумме внешнего сопротивления r и внутреннего сопротивления источника ЭДС. Обобщением закона Ома на случай разветвлённой цепи является правило 2-е Кирхгофа.

Закон Ома можно записать в дифференциальной форме, связывающей в каждой точке проводника плотность тока j с полной напряжённостью электрического поля. Потенциальное. электрическое поле напряжённости Е, создаваемое в проводниках микроскопическими зарядами (электронами, ионами) самих проводников, не может поддерживать стационарное движение свободных зарядов (ток), т. к. работа этого поля на замкнутом пути равна нулю. Ток поддерживается неэлектростатическими силами различного происхождения (индукционного, химического, теплового и т.д.), которые действуют в источниках ЭДС и которые можно представить в виде некоторого эквивалентного непотенциального поля с напряженностьюEСТ, называемого сторонним. Полная напряженность поля, действующего внутри проводника на заряды, в общем случае равна E+EСТ. Соответственно, дифференциальный закон Ома имеет вид:

или , (4)

где — удельное сопротивление материала проводника, а — его удельная электропроводность.

Закон Ома в комплексной форме справедлив также для синусоидальных квазистационарных токов:

(5)

где zполное комплексное сопротивление: , r– активное сопротивление, а x — реактивное сопротивление цепи. При наличии индуктивности L и емкости С в цепи квазистационарного тока частоты

.
3.Виды закона Ома.

Существует несколько видов закона Ома.

Закон Ома для однородного участка цепи (не содержащего источника тока): сила тока в проводнике прямо пропорциональна приложенному напряжению и обратно пропорциональна сопротивлению проводника:

Закон Ома для замкнутой цепи: сила тока в замкнутой цепи равна отношению ЭДС источника тока к суммарному сопротивлению всей цепи:

где R — сопротивление внешней цепи, r – внутреннее сопротивление источника тока.

R — +
R

Закон Ома для неоднородного участка цепи (участка цепи с источником тока):

R

;

где — разность потенциалов на концах участка цепи, — ЭДС источника тока, входящего в участок.

Способность вещества проводить ток характеризуется его удельным сопротивлением либо проводимостью . Их величина определяется химической природой вещества и условиями, в частности температурой, при которых оно находится. Для большинства металлов удельное сопротивление растет с температурой приблизительно по линейному закону:

;

где — удельное сопротивление при 0°С, t — температура по шкале Цельсия, а — коэффициент, численно равный примерно 1/273. Переходя к абсолютной температуре, получаем
При низких температурах наблюдаются отступления от этой закономерности. В большинстве случаев зависимость от T следует кривой 1 на рисунке.

Величина остаточного сопротивления в сильной степени зависит от чистоты материала и наличия остаточных механических напряжений в образце. Поэтому после отжига заметно уменьшается. У абсолютно чистого металла с идеально правильной кристаллической решеткой при абсолютном нуле .

У большой группы металлов и сплавов при температуре порядка нескольких градусов Кельвина сопротивление скачком обращается в нуль (кривая 2 на рисунке). Впервые это явление, названное сверхпроводимостью, было обнаружено в 1911 г. Камерлинг — Оннесом для ртути. В дальнейшем сверхпроводимость была обнаружена у свинца, олова, цинка, алюминия и других металлов, а также у ряда сплавов. Для каждого сверхпроводника имеется своя критическая температура Тк, при которой он переходит в сверхпроводящее состояние. При действии на сверхпроводник магнитного поля сверхпроводящее состояние нарушается. Величина критического поля HK, разрушающего сверхпроводимость, равна нулю при Т = Тки растет с понижением температуры.

Полное теоретическое объяснение сверхпроводимости было дано в 1958 г. советским физиком Н. Н. Боголюбовым и его сотрудниками.

Зависимость электрического сопротивления от температуры положена в основу термометров сопротивления. Такой термометр представляет собой металлическую (обычно платиновую) проволоку, намотанную на фарфоровый или слюдяной каркас. Проградуированный по постоянным температурным точкам термометр сопротивления позволяет измерять с точностью порядка нескольких сотых градуса как низкие, так и высокие температуры.
4.Краткая биография Кирхгофа.

Кирхгоф Густав Роберт(12.03.1824 – 17.10.1887) — немецкий физик.

Густав Кирхгоф родился в Кенигсберге в семье юриста. Окончив гимназию, он поступил в Кенигсбергский университет. После окончания университета Кирхгоф некоторое время преподавал в Берлине. В 1854 г. по совету химика Р. Бунзена Кирхгофа приглашают в Гейдельбергский университет, с которым связаны многие годы его небогатой внешними событиями жизни. Лишь безвременная смерть жены, оставившей ему четверых детей, и нелепый случай, сделавший его калекой, вынужденным передвигаться на кресле или с костылями, нарушили размеренный ход его жизни. Л. Больцман писал об этом так: «В жизни Кирхгофа не было ничего выдающегося, что соответствовало бы необычности его гения. Его жизнь была обычной жизнью немецкого профессора университета. Великие события происходили исключительно в его голове».

В начале своей научной деятельности Кирхгоф еще студентом начал исследовать законы распространения тока в электрических цепях. В 1849 г. он сформулировал свои знаменитые правила, которые до сих пор применяются для их расчета. В 1857 г. он опубликовал работу о распространении переменных токов по проводам, некоторые выводы которой предвосхитили теорию электромагнитного поля Максвелла.

Однако главный цикл работ Кирхгофа в Гейдельберге – анализ спектров излучения. Еще в 1855 г., сразу по приезде в Гейдельберг, Кирхгоф присоединился к исследованиям Р.Бунзена, который пытался установить химический состав солей по цвету пламени горелки (сейчас она так и называется бунзеновской горелкой). Кирхгоф сразу же заметил, что значительно более эффективным тестом на наличие того или иного вещества является анализ линейчатых спектров испускания этих веществ. Он сконструировал с помощью Бунзена новый, более совершенный, чем прежние, призматический спектроскоп и определил линии спектров множества элементов, открыв в процессе работы новые элементы — цезий и рубидий. Таким образом, Кирхгоф и Бунзен могут считаться создателями спектрального анализа, т.е. экспериментальной основы современной астрофизики.

Работа Кирхгофа по изучению спектров испускания элементов привела его к заключению о связи между спектральными линиями и темными линиями, обнаруженными Фраунгофером в спектре Солнца. Кирхгоф показал, что знаменитая желтая D-линия в спектре испускания натрия точно соответствует двум темным линиям в солнечном спектре. Это привело его к выводу, что атмосфера Солнца содержит натрий и этот натрий поглощает из непрерывного спектра солнечного излучения как раз ту часть, которая имеет длину волны, равную длине волны D-линии. Свою догадку Кирхгоф проверил в лаборатории, имитировав солнечный свет светом бунзеновской горелки и внеся в ее пламя поваренную соль.

Изучение связи между испусканием и поглощением излучения привело Кирхгофа к исследованию излучения нагретых тел. В 1862 г. он ввел понятие абсолютно черного тела и в качестве идеального излучателя (модель черного тела) предложил металлическую нагретую полость с маленькой дырочкой. Он сформулировал, опираясь на термодинамические соотношения, важнейший закон излучения черного тела: спектр излучения является универсальной функцией длины волны и температуры. Исследование излучения абсолютно черного тела стало на несколько десятилетий одной главных задач экспериментаторов и головной болью для теоретиков, так как классическая физика не могла объяснить наблюдаемого спектра излучения. Загадку излучения абсолютно черного тела разрешил в 1900 г. своей гипотезой квантов М.Планк, ученик Г.Кирхгофа. Поэтому без преувеличения можно сказать, что Кирхгоф внес важный вклад и в создание квантовой механики.

Кирхгоф был прекрасным лектором, хотя иногда немного суховатым. Правда, затем он великолепно редактировал и издавал свои лекции по разным вопросам теоретической физики. Многие германские физики в течение нескольких десятилетий учились по изданным лекциям Кирхгофа.

Несмотря на болезнь, лишавшую его подвижности, Кирхгоф был неутомимым исследователем и живым, интересным собеседником, любившим шутку. Однажды его банкир, под впечатлением от рассказов Кирхгофа о возможности определить химический состав Солнца, спросил: «Какой толк в том, что на Солнце есть золото, если я не могу перенести его на Землю и пощупать?» Несколькими годами спустя Кирхгоф получил золотую медаль Лондонского королевского общества и большую премию в золотых соверенах за свои исследования. Он вызвал своего банкира и передал ему свою награду, заметив шутливо: «Вот ваше золото из Солнца!»
5.Общий вид законов Кирхгофа.

Законы Кирхгофа являются одной из форм закона сохранения энергии и потому относятся к фундаментальным законам природы.

Середина XIX века как раз стала временем активных исследований свойств электрических цепей, и результаты этих исследований быстро находили практические применения. Базовые правила расчета простых цепей, такие как закон Ома, были уже достаточно хорошо проработаны. Проблема состояла в том, что из проводов и различных элементов электрических цепей технически уже можно было изготовлять весьма сложные и разветвленные сети — но никто не знал, как смоделировать их математически, чтобы рассчитать их свойства. Кирхгофу удалось сформулировать правила, позволяющие достаточно просто анализировать самые сложные цепи, и законы Кирхгофа до сих пор остаются важным рабочим инструментом специалистов в области электронной инженерии и электротехники.

Оба закона Кирхгофа формулируются достаточно просто и имеют понятную физическую интерпретацию. Первый закон гласит, что если рассмотреть любой узел цепи (то есть точку разветвления, где сходятся три или более проводов), то сумма поступающих в цепь электрических токов будет равна сумме исходящих, что, вообще говоря, является следствием закона сохранения электрического заряда. Например, если вы имеете Т-образный узел электрической цепи и по двум проводам к нему поступают электрические токи, то по третьему проводу ток потечет в направлении от этого узла, и равен он будет сумме двух поступающих токов. Физический смысл этого закона прост: если бы он не выполнялся, в узле непрерывно накапливался бы электрический заряд, а этого никогда не происходит.

Второй закон не менее прост. Если мы имеем сложную, разветвленную цепь, ее можно мысленно разбить на ряд простых замкнутых контуров. Ток в цепи может различным образом распределяться по этим контурам, и сложнее всего определить, по какому именно маршруту потекут токи в сложной цепи. В каждом из контуров электроны могут либо приобретать дополнительную энергию (например, от батареи), либо терять ее (например, на сопротивлении или ином элементе). Второй закон Кирхгофа гласит, что чистое приращение энергии электронов в любом замкнутом контуре цепи равно нулю. Этот закон также имеет простую физическую интерпретацию. Если бы это было не так, всякий раз, проходя через замкнутый контур, электроны приобретали или теряли бы энергию, и ток бы непрерывно возрастал или убывал. В первом случае можно было бы получить вечный двигатель, а это запрещено первым началом термодинамики; во втором — любые токи в электрических цепях неизбежно затухали бы, а этого мы не наблюдаем.

Самое распространенное применение законов Кирхгофа мы наблюдаем в так называемых последовательных и параллельных цепях.

В последовательной цепи (яркий пример такой цепи — елочная гирлянда, состоящая из последовательно соединенных между собой лампочек) электроны от источника питания по серии проводов последовательно проходят через все лампочки, и на сопротивлении каждой из них напряжение падает согласно закону Ома.

В параллельной цепи провода, напротив, соединены таким образом, что на каждый элемент цепи подается равное напряжение от источника питания, а это означает, что в каждом элементе цепи сила тока своя, в зависимости от его сопротивления. Пример параллельной цепи является — ламп «лесенкой»: напряжение подается на шины, а лампы смонтированы на поперечинах. Токи, проходящие через каждый узел такой цепи, определяются по второму закону Кирхгофа.
6.Виды законов Кирхгофа.

Первый закон Кирхгофа является следствием принципа непрерывности электрического тока, в соответствии с которым суммарный поток зарядов через любую замкнутую поверхность равен нулю, т.е. количество зарядов выходящих через эту поверхность должно быть равно количеству входящих зарядов. Основание этого принципа очевидно, т.к. при нарушении его электрические заряды внутри поверхности должны были бы либо исчезать, либо возникать без видимых причин.

Если заряды перемещаются внутри проводников, то они образуют в них электрический ток. Величина электрического тока может измениться только в узле цепи, т.к. связи считаются идеальными проводниками. Поэтому, если окружить узел произвольной поверхностью s (рис. 1), то потоки зарядов через эту поверхность будут тождественны токам в проводниках образующих узел и
суммарный ток в узле должен быть равным нулю.

Для математической записи этого закона нужно принять систему обозначений направлений токов по отношению к рассматриваемому узлу. Можно считать токи направленные к узлу положительными, а от узла отрицательными. Тогда для узла рис. 1 уравнение Кирхгофа будет иметь вид I3+I4I1I2 = 0 или I3+I4=I1+I2 .

Обобщая сказанное на произвольное число ветвей сходящихся в узле, можно сформулировать первый закон Кирхгофа следующим образом:


  • алгебраическая сумма токов в любом узле электрической цепи равна нулю



(1)

или

  • в любом узле сумма токов направленных к узлу равна сумме токов направленных от узла

, где p+q=n.

(2)

Очевидно, что обе формулировки равноценны и выбор формы записи уравнений может быть произвольным. Существенным является только соглашение о знаках токов для данной цепи, т.е. в пределах описания одной электрической цепи нельзя для разных узлов использовать разные знаки для токов направленных к узлам или от узлов.

При составлении уравнений по первому закону Кирхгофа направления токов в ветвях электрической цепи выбирают обычно произвольно. При этом необязательно даже стремиться, чтобы во всех узлах цепи присутствовали токи разных направлений. Может получиться так, что в каком-либо узле все токи сходящихся в нем ветвей будут направлены к узлу или от узла, нарушая тем самым принцип непрерывности. В этом случае в процессе определения токов один или несколько из них окажутся отрицательными, что будет свидетельствовать о протекании их в направлении противоположном принятому.

Второй закон Кирхгофа связан с понятием потенциала электрического поля, как работы, совершаемой при перемещении единичного точечного заряда в пространстве. Если такое перемещение совершается по замкнутому контуру, то суммарная работа при возвращении в исходную точку будет равна нулю. В противном случае путем обхода контура можно было бы получать положительную энергию, нарушая закон ее сохранения.

Каждый узел или точка электрической цепи обладает собственным потенциалом и, перемещаясь вдоль замкнутого контура, мы совершаем работу, которая при возврате в исходную точку будет равна нулю. Это свойство потенциального электрического поля и описывает второй закон Кирхгофа в применении к электрической цепи.

Он также как и первый закон формулируется в двух вариантах, связанных с тем, что падение напряжения на источнике ЭДС численно равно электродвижущей силе, но имеет противоположный знак. Поэтому, если какая либо ветвь содержит сопротивление и источник ЭДС, направление которой согласно с направлением тока, то при обходе контура эти два слагаемых падения напряжения будут учитываться с разными знаками. Если же падение напряжения на источнике ЭДС учесть в другой части уравнения, то его знак будет соответствовать знаку напряжения на сопротивлении.

Сформулируем оба варианта второго закона Кирхгофа, т.к. они принципиально равноценны:


  • алгебраическая сумма падений напряжения вдоль любого замкнутого контура электрической цепи равна нулю



(3)

Примечание: знак + выбирается перед падением напряжения на резисторе, если направление протекания тока через него и направление обхода контура совпадают; для падений напряжения на источниках ЭДС знак + выбирается, если направление обхода контура и направление действия ЭДС встречны независимо от направления протекания тока;

  • алгебраическая сумма ЭДС вдоль любого замкнутого контура равна алгебраической сумме падений напряжения на резисторах в этом контуре

, где p+q=n

(4)

Примечание: знак + для ЭДС выбирается в том случае, если направление ее действия совпадает с направлением обхода контура, а для напряжений на резисторах знак + выбирается, если в них совпадают направление протекания тока и направление обхода.

Здесь также как и в первом законе оба варианта корректны, но на практике удобнее использовать второй вариант, т.к. в нем проще определить знаки слагаемых.

С помощью законов Кирхгофа для любой электрической цепи можно составить независимую систему уравнений и определить любые неизвестные параметры, если число их не превышает число уравнений. Для выполнения условий независимости эти уравнения должны составляться по определенным правилам.

Общее число уравнений N в системе равно числу ветвей Nв минус число ветвей, содержащих источники тока NJ , т.е. N = Nв — NJ .

Наиболее простыми по выражениям являются уравнения по первому закону Кирхгофа, однако их число N1не может быть больше числа узлов Nу минус один.

Недостающие уравнения составляются по второму закону Кирхгофа, т.е.


N1 = Nу -1 ;

(5)

N2 = N — N1 = Nв — NJ — N1.

(6)

Сформулируем алгоритм составления системы уравнений по законам Кирхгофа :


  1. определить число узлов и ветвей цепи Nу и Nв;

  2. определить число уравнений по первому и второму законам N1 и N2.;

  3. для всех ветвей (кроме ветвей с источниками тока) произвольно задать
    направления протекания токов;

  4. для всех узлов, кроме одного, выбранного произвольно, составить уравнения по первому закону Кирхгофа;

  5. произвольно выбрать на схеме электрической цепи замкнутые контуры таким образом, чтобы они отличались друг от друга по крайней мере одной ветвью и чтобы все ветви, кроме ветвей с источниками тока, входили по крайней мере в один контур;

  6. произвольно выбрать для каждого контура направление обхода и составить уравнения по второму закону Кирхгофа, включая в правую часть уравнения ЭДС действующие в контуре, а в левую падения напряжения на резисторах. Примечание: Знак ЭДС выбирают положительным, если направление ее действия совпадает с направлением обхода независимо от направления тока; а знак падения напряжения на резисторе принимают положительным, если направление тока в нем совпадает с направлением обхода.

Рассмотрим этот алгоритм на примере рис 2.

Здесь светлыми стрелками обозначены выбранные произвольно направления токов в ветвях цепи. Ток в ветви с R4 не выбирается произвольно, т.к. в этой ветви он определяется действием источником тока.

Число ветвей цепи равно 5, а т.к. одна из них содержит источник тока, то общее число уравнений Кирхгофа равно четырем.

Число узлов цепи равно трем (a, b и c), поэтому число уравнений по первому закону Кирхгофа равно двум и их можно составлять для любой пары из этих трех узлов. Пусть это будут узлы a и b, тогда


a) J + IE1 = J + IR1 =IR3J + IR1 IR3 = 0

(7)

b) IR3 + IE2 = IR1 + IR2IR3 + IE2IR1IR2 = 0

(8)

По второму закону Кирхгофа нужно составить два уравнения. Выберем два контура I и II так, чтобы все ветви, кроме ветви с источником тока попали по крайней мере в один из них, и зададим произвольно направление обхода как показано стрелками. Тогда

I) —E1 = IR1R1 + IR3R3

(9)

II) E2 = IR2R2

(10)

При выборе контуров и составлении уравнений все ветви с источниками тока должны быть исключены, т.е. контуры обхода не должны включать ветви с источниками тока. Это не означает что для контуров с источниками тока нарушается второй закон Кирхгофа. Просто при необходимости определения падения напряжения на источнике тока или на других элементах ветви с источником тока это можно сделать после решения системы уравнений. Например, на рис. 2 можно создать замкнутый контур из элементов R3, R4 , J и E2, и для него будет справедливым уравнение

IR3R3+ E2+JR4+ UJ = 0 ,

где UJ — падение напряжения на источнике тока J.

Из сказанного выше очевидно, что законы Кирхгофа необязательно использовать в виде систем уравнений. Они справедливы всегда для любого узла и для любого замкнутого контура любой электрической цепи.
Современные средства математического анализа позволяют легко получить результат решения составленной выше системы уравнений, если она записана в матричной форме AX=B. Это можно сделать, например, для токов в качестве неизвестных.

Каждая строка матрицы A должна соответствовать одному из уравнений (7)-(10). Поэтому в строки матрицы A нужно включить все коэффициенты при токах соответствующего уравнения, в той последовательности, в какой эти токи включены в координаты вектора неизвестных величин. Если какой-либо ток отсутствует в уравнении, то в качестве элемента матрицы нужно указать нуль. Для включения в матрицу уравнения по первому закону Кирхгофа удобнее записывать в форме (1) с нулевой правой частью, однако, для уравнения (7) нужно перенести ток источника J в правую часть, т.к. он не входит в число неизвестных.

Вектор неизвестных токов X представляет собой столбец, в который включены неизвестные токи в произвольной последовательности.

Вектор B представляет собой столбец, координатами которого являются источники электрической энергии, действующие в цепи (правая часть уравнений (7)-(10)). Порядок включения их в столбец должен соответствовать порядку записи уравнений в строки матрицы A .

Составим матричное уравнение для схемы рис. 2, используя полученные ранее уравнения (7)-(8) и (9)-(10).

Здесь для упрощения восприятия строки записи помечены указателями на тот узел или контур, которому они соответствуют.

Список использованной литературы:
Прохоров А. М. Физический энциклопедический словарь, М., 1983

Дорфман Я. Г. Всемирная история физики. М., 1979
Ом Г. Определение закона, по которому металлы проводят контактное электричество. – В кн.: Классики физической науки. М., 1989

Роджерс Э. Физика для любознательных, т. 3. М., 1971
Орир Дж. Физика, т. 2. М., 1981
Джанколи Д. Физика, т. 2. М., 1989

ЗАКОНЫ ОМА И КИРХГОФА. ПОТЕНЦИАЛЬНАЯ ДИАГРАММА ЭЛЕКТРИЧЕСКОЙ ЦЕПИ

Лабораторная работа № 3

ЗАКОНЫ ОМА И КИРХГОФА. ПОТЕНЦИАЛЬНАЯ ДИАГРАММА ЭЛЕКТРИЧЕСКОЙ ЦЕПИ

Краткое содержание работы

 В процессе выполнения работы измеряют:

-силу тока, напряжения на элементах в неразветвленной цепи и проверяют закон Ома;

— силу тока, напряжения на элементах в сложной цепи и проверяют законы Кирхгофа;

-проверяют принципы суперпозиции для линейной цепи.

Аналитически рассчитывают силу токов в обеих цепях и сравнивают с опытными данными. Строят потенциальные диаграммы для одного и того же контура при  разных силах токов в его элементах. Аналитически определяют потенциалы в цепи и сравнивают с опытными данными.

Подготовка к работе

  1.  Как формулируются первый и второй законы Кирхгофа?
  2.  Почему при расчете цепи число независимых уравнений, составляемых по первому закону Кирхгофа, на единицу меньше числа узлов?
  3.  Сколько независимых уравнений необходимо составлять по второму закону Кирхгофа?
  4.  Как проверить, удовлетворяют ли полученные опытным путем силы токов и напряжения соответственно первому и второму законам Кирхгофа?
  5.  Что такое потенциальная диаграмма контура электрической цепи?
  6.  Как построить потенциальную диаграмму по результатам опыта?
  7.  Как определить из потенциальной диаграммы силу и направление тока в ветви контура?
  8.  Как определить из потенциальной диаграммы значение и знак напряжения между любыми двумя точками электрической цепи?
  9.  Что такое принцип суперпозиции для линейной цепи и как он формулируется и проверяется?

10. Как повлияет на вид потенциальной диаграммы выбор другой точки с нулевым потенциалом? Привести аргументы.

Рабочее задание

  1.  В схеме, изображенной на рис.3.1, задать ЭДС Е1 и E2 источников энергии и их внутренние сопротивления R01 и R02, а также номиналы элементов цепи в соответствии вариантом задания, приведенного в табл.3.2. В качестве отсчетного потенциала, выбрать потенциал указанной точки.
  2.  Измерить силу токов, напряжения между именованными точками цепи и потенциалы этих же точек по значению и знаку. Рассмотреть три варианта состояния цепи в зависимости от значений источников напряжения. Результаты измерений свести в табл.3.1.

Примечания: 

А) Так как непосредственно измерить потенциал какой-нибудь точки принципиально невозможно, то всегда измеряют разность потенциалов между исследуемой точкой и той точкой, потенциал которой условно принят равным нулю (отсчетная точка для потенциалов).

Б) При всех измерениях  следует учитывать не только значение, но и знак разности потенциалов.

3. По известным из опыта п.2 напряжениям, ЭДС и сопротивлениям вычислить силу токов на участке цепи, не содержащем ЭДС, на участке цепи с ЭДС и в неразветвленной цепи в целом и проверить справедливость закона Ома во всех этих случаях.

  1.  Вычислить аналитически, с использованием метода уравнений Кирхгофа, силу и направления токов в ветвях для исходных вариантов  схем, а также напряжения на элементах.  Занести их в табл.3.1 и сравнить с измеренными значениями при опыте.
  2.  По данным опыта п.2 построить потенциальные диаграммы внешнего контура цепи для всех трех вариантов цепи (см.табл. 3.1) на одном графике.
  3.  По данным табл. 3.1 проверить соблюдение принципа суперпозиции и привести примеры.  
  4.  Сделать выводы по работе.

Таблица 3.1.

Варианты цепей

Исходный

Е1=0

Е2=0

опыт

рассч

опыт

рассч

опыт

рассч

Uaб

Uбв

Uвг

Uгд

Uдж

Uжа

Ua

I1

I2

I3

IR01

IR02

IR7

IR6

табл. 3.2 (напряжения  В,  сопротивления Ом )

№ вар

Е1

Е2

R01

R02

R1

R2

R3

R4

R5

R6

R7

ЗАЗ

узел

1

1

2

1

2

3

5

4

7

10

4

6

а

2

10

5

6

3

4

3

5

5

3

6

4

б

3

3

8

2

3

8

2

10

7

4

3

5

в

4

15

9

4

6

3

6

12

8

6

12

8

г

5

15

20

7

3

12

10

8

3

10

15

12

д

6

30

15

10

12

6

9

12

15

7

14

7

ж

7

25

40

5

7

15

5

20

10

9

15

10

а

8

50

20

3

5

10

15

8

12

7

6

3

б

9

40

15

2

5

18

9

15

20

15

4

7

г

10

20

35

5

12

20

15

9

25

15

5

9

д

Методические указания

Законы Кирхгофа являются основными законами электрической цепи.

Первый закон Кирхгофа, применительно к узлам электрической цепи, основан на принципе непрерывности электрического тока: Алгебраическая сумма токов, сходящихся и расходящихся в узле электрической цепи, равна нулю.

Второй закон Кирхгофа применяется к контурам электрической цепи и основан на принципе потенциальности электрического поля: Алгебраическая сумма падений напряжений во всех ветвях любого замкнутого контура электрической цепи равна алгебраической сумме ЭДС источников энергии, действующих в этом контуре.

При записи уравнений согласно законам  Кирхгофа необходимо соблюдать правило знаков для токов и падений напряжений на элементах цепи. Для первого закона достаточно считать токи одного какого-либо направления, по отношению к узлу, одинакового знака. Например, все токи, идущие от узла считать положительными, а к узлу – отрицательными.

При использовании второго закона Кирхгофа обычно поступают так:

А) Произвольно выбирают некоторое направление обхода замкнутого контура и это направление считают положительным;

Б) Если при обходе внутри  контура, мы проходим источник ЕДС от отрицательного полюса к положительному, то перед значением ЭДС этого источника в уравнении следует ставить знак «+», в противном случае « — »;

В) Перед значением падения напряжения на участке контура следует ставить знак «+», если условно выбранное направление тока  в элементе цепи совпадает с выбранным направлением обхода контура, в противном случае (направления противоположны) ставиться  « — ».

Принцип суперпозиции (наложения) позволяет разделять анализ линейной цепи, в которой действует два и более источников энергии , на более простые варианты анализа цепи в которой действует только один источник.

Принцип суперпозиции заключается в том, что ток  в любой ветви линейной цепи с несколькими источниками электрической энергии, равен алгебраической сумме токов, которые обусловлены в этой ветви каждым из источников отдельно.

В соответствии с принципом суперпозиции ток в любой ветви электрической линейной цепи может быть представлен:

                             Iк=Gк1E1 + Gк2E2 + …+ GкnEк + …+ GкмEм = Iк1 + Iк2 +…+ Iкn +…+Iкм

Где: Е1, Е2 …Ем – источники напряжений в ветвях цепи;  Gкм =1/Rkm – взаимная проводимость «к» и «м» ветви;  Gкn =1/Rkk – входная проводимость «к»– ветви;   Iкм – ток в ветви  «к», обусловленный источником  Eм в ветви  «м».   

 Диаграмму строят следующим образом:

  1.  Выбирают направление обхода замкнутого контура, для которого строят потенциальную диаграмму.
  2.  Обозначают буквами или цифрами точки контура.
  3.  Выбирают масштаб для сопротивлений и масштаб для потенциалов.
  4.  На оси абсцисс откладывают точки, отвечающие значениям сопротивлений    участков    контура    в    последовательности, соответствующей расположению их в схеме.
  5.  Из этих точек восстанавливают ординаты,  равные потенциалам точек электрической цепи.
  6.  Полученные точки потенциалов соединяют прямыми линиями.

При построении диаграмм удобно внутренние сопротивления источников ЭДС представить последовательно включенными с идеальным источником напряжения. Изменение потенциала вдоль сопротивления, обтекаемого током, на диаграмме изображается прямой линией, тангенс угла наклона которой к оси абсцисс пропорционален силе тока.

При определении потенциала необходимо учитывать следующее.

  1.  По направлению, совпадающему с направлением тока, в сопротивлении   происходит   понижение   потенциала,   а   в противоположном направлении — повышение.
  2.  При переходе через источник ЭДС в направлении от отрицательного зажима к положительному потенциал повысится на значение ЭДС источника и уменьшится на значение падения напряжения на внутреннем сопротивлении.

            а         R4      б         R5  в

        E1            E2

                                                                         I3

 R6          R7

   R01   R3       R02

                                                                    I1                          I2

              ж  R1      д   R2               г

Рис 3. 1.

Закон Кирхгофа о цепях — Electronics-Lab.com

Введение

В этом руководстве мы представляем один из самых фундаментальных и важных наборов законов для электрических цепей. Эти законы получили название и были установлены немецким врачом Густавом Кирхгофом в 1845 году.

Как и многие другие физические законы, законы схем Кирхгофа (KCL) относительно легко понять и вытекают из наблюдения закона сохранения энергии , который, вероятно, является одним из старейших и наиболее фундаментальных принципов физики.

Тем не менее, KCL могут быть простыми и доступными для понимания, они остаются основным инструментом, которым нужно овладеть для анализа схем, и до сих пор широко используются.

KCL состоит из двух различных законов относительно физических величин, которые определяют энергию в электрической цепи: ток, и напряжение. Далее мы отдельно представляем в двух разделах, текущие законы Кирхгофа и законы напряжения .

Перед этими разделами стоит представить в первом разделе структуру, в которой используются законы KCL, и многие определения, связанные с этим принципом.

В третьем разделе показан пример того, как применить KCL к реальной схеме и решить проблему с неизвестными параметрами.

Наконец, в последнем разделе кратко представлены ограничения KCL для некоторых частных случаев.

Структура и определения

Основа для применения законов Кирхгофа — это электрических цепей , которые состоят из источника питания или генератора , оба полюса которого соединены с промежуточным звеном компонентов в замкнутой конфигурации .Электрические цепи нарисованы в соответствии с моделью с сосредоточенными элементами , которая предполагает, что компоненты являются идеальными, и соответствующее представление показано на , рис. 1 ниже:

Рис.1: Модель электрической цепи с сосредоточенными элементами

Мы можем дать некоторые конкретные детали относительно топологии схем. Прямые линии представляют собой идеальные соединения / провода между различными элементами схем, что означает, что они не проявляют резистивного или реактивного поведения, следовательно, отсутствуют явления потери мощности или фазового сдвига.

Источник обеспечивает схему энергией, которая состоит из сигналов напряжения и тока (постоянного или переменного тока). Компоненты пассивные , они состоят из комбинации резистора, конденсатора и катушки индуктивности. Они могут быть подключены параллельно, как в Рисунок 1 , или последовательно. Активные компоненты, такие как усилители, не рассматриваются в этом руководстве, поскольку с ними связан внешний источник питания. Группа компонентов, соединенных на обоих выводах проводами, называется ответвлением .

Два важных топологических определения важны для того, чтобы впоследствии полностью понять законы Кирхгофа: узлов и петель . Узлы представляют собой соединения между ветвями, и они выделены на рис. 1 синими кружками. Петли выделены красными круговыми стрелками на предыдущем рисунке и представляют собой замкнутый путь ветвей.

Действующий закон Кирхгофа

Текущий закон также известен как узловой или переходный закон и утверждает, что алгебраическая сумма токов, встречающихся в узле, равна нулю .Простой пример можно проиллюстрировать узлом, соединяющим три ветви:

рис. 2: Узловое соединение трех ветвей

Таким образом, закон гласит, что сумма токов, входящих в узел, равна току (токам) на выходе из соединения. В нашем примере это предложение математически переводится в I 1 + I 2 = I 3 или I 1 + I 2 -I 3 = 0 , текущий I 3 отрицательный, потому что он выходит из узла.

В общем случае узловое соединение из N ветвей, токи которых обозначены как I 1 , I 2 ,…, I N , удовлетворяет следующему Уравнению 1 :

уравнение 1: Текущий закон Кирхгофа

Знаковая функция sign равна +1 , если текущий I k входит в узел, или -1 , если он существует.

Узловой закон прямо записан из наблюдения, что заряд в замкнутой системе инвариантен.Это предположение также известно как принцип сохранения заряда .

В физике принцип — это наблюдение, которое ни один опыт не подтвердил, но не продемонстрировал; это эквивалент постулатов в математике.

Закон напряжения Кирхгофа

Закон напряжения также известен как правило петли , он очень похож на узловое правило, но применяется к петлям, а не к узлам. Этот второй закон гласит, что алгебраическая сумма напряжений в контуре цепи равна нулю. Простой пример можно проиллюстрировать с источником постоянного тока, питающим последовательный RC-фильтр:

рис. 3: Петля с тремя различными напряжениями

Знак напряжений определяется направлением стрелок, обычно источник считается положительным, так что стрелки по часовой стрелке являются положительными, а против часовой стрелки — отрицательными. Таким образом, закон Кирхгофа утверждает, что В S = В R + V C или В S -V R -V C = 0 .

Для контура с генерацией N и падением напряжений V 1 , V 2 ,…, V N , Уравнение 2 выполняется:

уравнение 2: Закон Кирхгофа для напряжения

Знаковая функция sgn равна +1 , когда генерируется напряжение (источник в нашем примере), или -1 , когда наблюдается падение (с пассивными компонентами в Рисунок 3 ).

Заявление KCL

В этом разделе мы показываем процесс решения типичных проблем, которые можно решить с помощью KCL.Рассмотрим три источника S 1 , S 2 и S 3 , подключенных к резисторам R 1 , R 2 и R 3 в конфигурации, показанной на Рисунок 4 :

рис 4: Пример конфигурации схемы

Источники постоянного тока идеальны, что означает отсутствие внутреннего сопротивления. Возьмем S 1 = 4 В, S 2 = 3 В, S 3 = 10 В и R 1 = 3 Ом, R 2 = 2 Ом, R 3 = 1 Ом. .

Из текущего закона Кирхгофа мы можем записать следующие равенства для узлов 1 и 2:

  • Узел 1: I 1 = I 2 + I 3
  • Узел 2: I 2 + I 3 = I 1 , что аналогично уравнению узла 1

Из закона напряжения Кирхгофа запишем равенства для контуров 1 и 2:

  • Петля 1: S 1 = S 2 + R 2 × I 2 + R 1 × I 1
  • Петля 2: S 2 + R 2 × I 2 = S 3 + R 3 × I 3

Мы можем по-разному записать эти уравнения, чтобы получить следующую систему из 3 уравнений с 3 неизвестными параметрами: I 1 , I 2 и I 3 :

уравнение 3: Линейная система KCL уравнения

Эта система может быть решена методом исключения, который состоит в замене I 1 на I 2 + I 3 во второй строке (L2) и исключении члена I 3 путем сложения R 3 × L2 + R 1 × L3.

Мы находим непосредственно I 2 = 4,2 A , затем мы можем найти I 3 , заменяя I 2 в L3, что дает I 3 = 1,4 A и, наконец, мы получаем I 1 = I 2 + I 3 = 5,6 A .

Пределы KCL

В первом разделе мы рассмотрели структуру, в которой применяется KCL, но есть и другие, более тонкие условия, которые схема должна соблюдать. В этом разделе мы кратко выделим эти дополнительные условия для действия KCL.

Первое условие известно как квазистатическое приближение и состоит в том, что время распространения сигнала должно быть незначительным по сравнению с периодом сигнала, это дает условие на размеры схемы.

Например, рассмотрим сигнал переменного тока 200 кГц (T = 5 мкс), если приемник расположен в цепи на D = 10 см , время распространения будет Δt = D / c = 0,33 нс с c — скорость света. В этом случае Δt << T, квазистатическое приближение действительно и условие применения KCL соблюдается.

Однако, если приемник вместо этого расположен на D = 1 км , время распространения становится Δt = 3,3 мкс и неравенство Δt << T не соблюдается, поэтому приближение недействительно и KCL не может применяться к цепи. .

В квазистатическом приближении считается, что любое изменение источника немедленно распространяется в любой точке цепи, что позволяет избежать запаздывающих эффектов, которые могут сделать недействительным KCL.

Это утверждение может быть продемонстрировано с помощью уравнения Максвелла-Ампера , в котором вариативный член может быть исключен, когда квазистатическое приближение действительно, текущий закон Кирхгофа может быть продемонстрирован с помощью теоремы Грина-Остроградского .

Еще одно общее условие, придающее силу KCL, состоит в том, чтобы сказать, что вариациями магнитного потока в контурах цепи следует пренебречь. Согласно закону индукции изменения магнитного потока создают в цепи индуцированные токи и, следовательно, индуцированные напряжения.

Изменение магнитного потока делает недействительным правило петли, вводя новый термин напряжения, который объясняется не компонентами или топологией схемы, а внешним источником.

Заключение

KCL — это фундаментальные законы электроники, которые могут применяться к электрическим схемам, состоящим из петель и узлов . Эти топологические определения наряду с другими представлены в первом разделе статьи, в котором описывается структура, в которой применяется KCL.

Законы Кирхгофа состоят из закона тока и напряжения, который отражает принцип сохранения энергии в цепи.

Текущий закон учитывает сохранение заряда , он утверждает, что алгебраическая сумма токов в узле равна нулю.Закон напряжения гласит, что алгебраическая сумма напряжений в контуре равна нулю.

В следующем разделе показано, что использование обоих этих законов может решить типичные электронные проблемы путем решения систем линейных уравнений.

Наконец, мы кратко представляем в последнем разделе, что некоторые тонкие условия относительно размеров цепи и существования внешнего магнитного потока должны быть соблюдены для того, чтобы KCL был действительным.

Закон Кирхгофа по току и закон Кирхгофа по напряжению

Закон Кирхгофа

Существует несколько простых соотношений между токами и напряжениями различных ветвей электрической цепи.Эти отношения определяются некоторыми основными законами, которые известны как законы Кирхгофа или более конкретно законы Кирхгофа по току и напряжению. Эти законы очень полезны при определении эквивалентного электрического сопротивления или импеданса (в случае переменного тока) сложной сети и токов, протекающих в различных ветвях сети. Эти законы были впервые выведены Гуатов Роберт Кирхгоф , и поэтому эти законы также упоминаются как Законы Кирхгофа .

Текущий закон Кирхгофа

В электрической цепи ток течет рационально как электрическая величина.
Поскольку поток тока рассматривается как поток количества, в любой точке цепи полный ток входит, в точности равен полному току, выходящему из точки. Точку можно рассматривать в любом месте схемы.


Предположим, что точка находится на проводнике, по которому течет ток, тогда тот же самый ток пересекает точку, которая, альтернативно, может сказать, что ток входит в точку, покидает точку.Как мы уже говорили, точка может быть в любом месте цепи, поэтому она также может быть точкой соединения в цепи.

Итак, общее количество тока, поступающего в точку соединения, должно быть точно равно общему количеству тока, выходящего из соединения. Это самая основная вещь о протекании тока, и, к счастью, Закон Кирхгофа о течениях говорит то же самое. Этот закон также известен как Первый закон Кирхгофа , и этот закон гласил, что в любой точке соединения в электрической цепи сумма всех токов ответвления равна нулю.Если мы рассмотрим, что все токи, входящие в переход, считаются положительными токами, то условно все токи ответвления, выходящие из перехода, будут отрицательными. Теперь, если мы сложим все эти положительные и отрицательные токи со знаком, очевидно, мы получим нулевой результат.
Математическая форма Текущего закона Кирхгофа выглядит следующим образом:
У нас есть перекресток, где n пляжей сходятся вместе.
Lets,

Токи в ветвях 1, 2, 3…. м выходят на перекресток.
Тогда как токи в ответвлениях уходят от стыка.
Значит токи в ветвях 1, 2, 3…. m можно считать положительным согласно общему соглашению, и точно так же токи в ответвлениях можно рассматривать как отрицательные.
Следовательно, все токи ответвления относительно указанного перехода равны —

Теперь сумма всех токов в переходе —

Это равно нулю согласно Закону Кирхгофа о токах.
Следовательно,
Математическая форма Первого закона Кирхгофа : ∑ I = 0 на любом стыке электрической сети.

Видеопрезентация закона Кирхгофа — основная теория

Закон Кирхгофа о напряжении


Этот закон касается падений напряжения на различных ветвях электрической цепи. Подумайте об одной точке замкнутого контура в электрической цепи. Если кто-то перейдет к любой другой точке той же петли, он или она обнаружит, что потенциал в этой второй точке может отличаться от первой точки. Если он или она продолжит идти в какую-то другую точку цикла, он или она может найти в этом новом месте другой потенциал.Если он или она пойдет дальше по замкнутому циклу, в конечном итоге он или она достигнет начальной точки, с которой было начато путешествие. Это означает, что он или она возвращается к той же потенциальной точке после перехода через разные уровни напряжения. В качестве альтернативы можно сказать, что чистый выигрыш по напряжению и падение чистого напряжения в замкнутом контуре равны. Это то, что утверждает закон о напряжении Кирхгофа. Этот закон также известен как Второй закон Кирхгофа .

Если мы рассматриваем замкнутый контур условно, если мы считаем, что все приросты напряжения вдоль контура положительны, то все падения напряжения вдоль контура следует рассматривать как отрицательные.Сумма всех этих напряжений в замкнутом контуре равна нулю. Предположим, что n элементов, соединенных вплотную друг к другу, образуют замкнутый контур. Среди этих элементов схемы m элементов — это источник напряжения, а количество элементов — n — m, таких как резисторы.
Напряжения источников равны
И падение напряжения на резисторах соответственно,
Как сказано, что коэффициент усиления напряжения обычно считается положительным, а падения напряжения считаются отрицательными, напряжения вдоль замкнутого контура равны —

Теперь согласно Закон Кирхгофа о напряжении , сумма всех этих напряжений приводит к нулю.

Соответственно Второй закон Кирхгофа , ∑V = 0.

Применение законов Кирхгофа к цепям

Распределение тока в различных ветвях цепи можно легко определить, применив Закон Кирхгофа в разных узлах или точки соединения в цепи. После того, как применяется закон напряжения Кирхгофа , каждый возможный контур в схеме генерирует алгебраическое уравнение для каждого контура. Решая все эти уравнения, можно легко узнать различные неизвестные токи, напряжения и сопротивления в цепях.

Некоторые популярные условные обозначения, которые мы обычно используем при применении KVL
  1. Резистивные падения в контуре из-за тока, протекающего по часовой стрелке, следует рассматривать как положительные падения.
  2. Резистивные падения в контуре из-за тока, протекающего против часовой стрелки, следует рассматривать как отрицательные падения.
  3. ЭДС батареи, вызывающая протекание тока по часовой стрелке в контуре, считается положительной.
  4. ЭДС батареи, вызывающая протекание тока против часовой стрелки, называется отрицательной.

Приложения, ограничения и решенные примеры!

Законы Кирхгофа для цепей считаются основой любого анализа электрических цепей. Существует два типа законов Кирхгофа для цепей: закон Кирхгофа по току и закон Кирхгофа по напряжению. С помощью этих законов и уравнения для отдельных компонентов (резистора, конденсатора и катушки индуктивности) мы анализируем цепи. В 19 веке ученый по имени Густав Роберт Кирхгоф внес большой вклад в эту теорию и обеспечил лучшее понимание электрических цепей.Он также обнаружил, что ток течет в проводнике со скоростью света.

Чтобы получить подробную информацию о кинетической теории газов, кандидаты могут посетить связанную статью.

Терминология цепи

  1. Схема: Это замкнутый путь, по которому течет ток.
  2. Путь: Путь рассматривается как одна линия, состоящая из элементов схемы и источников.
  3. Узел: Узел определяется как терминал или соединение, в котором два или более элемента будут соединены вместе и, таким образом, будут иметь общую точку для более чем одной ветви.
  4. Ветвь : Ветвь состоит из таких элементов, как резисторы и источники, подключенные между двумя узлами.
  5. Цикл: Цикл — это замкнутый путь, в котором элементы подсчитываются только один раз.
  6. Сетка: Сетка — это открытый контур без каких-либо элементов.

Если элементы соединены последовательно, ток, протекающий через каждый из них, будет одинаковым. Если элементы соединены параллельно, напряжение на каждом компоненте остается неизменным.

Подробнее о расстоянии и смещении см. В связанной статье.

Густав Роберт Кирхгоф

Густав Роберт Кирхгоф, немецкий физик, родился 12 марта 1824 года. Он внес свой вклад в фундаментальное понимание электрических цепей, спектроскопии и излучения черного тела нагретыми объектами. В 19 веке Густав Роберт Кирхгоф внес большой вклад в эту теорию и обеспечил лучшее понимание электрических цепей.Он также обнаружил, что ток течет в проводнике со скоростью света.

Первый закон — Закон Кирхгофа (KCL)

Первый закон Кирхгофа касается тока в цепи. Согласно этому закону алгебраическая сумма токов в любом узле цепи равна нулю. Полный ток, поступающий в переход, в точности равен полному току, выходящему из перехода. Этот закон основан на сохранении сборов.

Из изображения ниже мы можем сказать, что:

\ ((i_2 + i_3 + i_5 + i_6 + i_7) = (i_1 + i_4 + i_8) \)

∑ incoming = ∑ outgoing

Вы также можете проверьте подробности о векторе.

В. Найдите ток в данной цепи.

Отв. В узле возможно только одно значение тока

Итак, здесь нарушение KCL

Итак, ток в цепи невозможен.

Проверьте артикул Uniform Circular Motion здесь.

Наконечник памяти

  • KCL применяется к любой сосредоточенной сети независимо от характера сети; односторонние или двусторонние, активные или пассивные, линейные или нелинейные.
  • KCL не применяется к распределенным сетям.
  • Ток всегда проходит менее устойчивым путем.
  • KCL всегда обеспечивает сохранение заряда.
  • Максимальный ток протекает при коротком замыкании из-за нулевого сопротивления.
  • В разомкнутой цепи нет протекания тока из-за бесконечного сопротивления.

Получите подробную информацию о вихревых токах и токах смещения.

Второй закон — закон напряжения Кирхгофа (KVL)

Закон напряжения Кирхгофа или второй закон касается падения напряжения в цепи.Когда в цепи протекает ток, величина тока изменяется в зависимости от произведения тока на сопротивление или ЭДС, с помощью которой он подключен в цепи. Согласно этому закону алгебраическая сумма напряжения (или падений напряжения) на любом замкнутом пути сети в определенном направлении равна нулю.

Знак при движении в цикле полностью зависит от пользователя,

Если мы рассмотрим переход от положительного (+) к отрицательному (-) (это зависит от пользователя), он действует как падение или прирост.

Этот знак не влияет на ответ.

Предположим, мы рассматриваем это как падение (-) при переходе от положительного (+) к отрицательному (-).

  • Начнем с сопротивления R1. Ток идет от одной точки к другой (от положительной к отрицательной), а падение напряжения считается отрицательным (-).
  • При переходе от ЭДС ток E2 меняется с положительного на отрицательный, и падение принимается как –E2
  • Аналогично для R2 и R3
  • Наконец, при переходе от E1 ток меняется с отрицательного на положительный, и здесь берется усиление.

-iR1 — E2 — iR2 + E1 = 0

E1 — E2 = iR1 + iR2

Q. Найдите ток I в данной цепи.

Отв. Применим KVL в данном контуре, тогда мы получим,

120-30 I — 2VA + VA = 0

120 = 30 I + VA …………… (1)

Также из закона Ома на выходе мы можно получить

ВА = — 15 I ……………… .. (2)

Решая уравнения (1) и (2), получаем

I = 8 ампер.

Итак, в цепи протекает ток 8А.

Проверьте питание в цепи переменного тока для получения подробной информации здесь.

Наконечник памяти

  • KVL применяется к любой сосредоточенной сети независимо от характера сети; односторонние или двусторонние, активные или пассивные, линейные или нелинейные.
  • KVL не распространяется на распределенные сети.
  • Падение напряжения при коротком замыкании равно нулю из-за нулевого сопротивления.
  • KVL всегда экономит энергию.
  • Максимальное напряжение возникает в разомкнутой цепи из-за бесконечного сопротивления.
  • Напряжение в параллельном тракте постоянное, а в последовательном — делится.

Применение закона Кирхгофа

С помощью закона Кирхгофа можно найти:

  1. Значения тока, напряжения и внутреннего сопротивления в цепях постоянного тока.
  2. Применяя этот закон, мы также можем найти неизвестное сопротивление в цепи.
  3. Мост Уитстона — важное приложение закона Кирхгофа. Он используется в анализе сетки и узлов.

Ограничения закона Кирхгофа

  1. Законы KCL и KVL не подходят для цепей переменного тока высокой частоты.Текущий закон применяется только тогда, когда электрический заряд в цепи постоянный.
  2. Где KVL применяется в предположении, что магнитные поля не изменяются в замкнутой цепи. Таким образом, мы не можем применять KVL, когда магнитное поле изменяется внутри цепи.

Прочтите об атомах и ядрах здесь.

Законы Кирхгофа для цепей: краткое изложение

  1. Густав Кирхгоф дает лучшее понимание решения и применения электрических цепей.
  2. Первый закон Кирхгофа гласит, что полный ток, который входит в узел или переход, равен общему току или заряду, выходящему из узла.В его основе лежит принцип сохранения заряда. Это также известно как правило соединения.
  3. Второй закон Кирхгофа гласит, что сумма падений напряжения равна сумме повышений напряжения. Этот закон основан на сохранении энергии. Это также известно как правило цикла.

Мы надеемся, что приведенные выше примечания к законам Кирхгофа об округах помогли вам лучше понять предстоящие экзамены JEE и другие конкурсные экзамены. Попрактикуйтесь прямо сейчас в приложении Testbook с помощью бесплатных пробных тестов.

Также подробно ознакомьтесь с типами термодинамических процессов, чтобы улучшить вашу подготовку.

Часто задаваемые вопросы о законе Кирхгофа

Q.1 Каков первый закон Кирхгофа?

Ans.1 Согласно этому закону алгебраическая сумма токов в любом узле цепи равна нулю. Полный ток, поступающий в переход, в точности равен полному току, выходящему из перехода. Этот закон основан на сохранении сборов.

Q.2 Что такое второй закон Кирхгофа?

Ans.2 Согласно этому закону алгебраическая сумма напряжения (или падений напряжения) на любом замкнутом пути сети в определенном направлении равна нулю.

Q.3 Что такое цепь?

Ans.3 Цепь — это замкнутая цепь, по которой протекает ток.

Q.4 Что такое петля?

Ans.4 Цикл — это замкнутый путь, в котором элементы подсчитываются только один раз.

Q.5 Что такое узел?

Ans.5 Узел определяется как терминал или соединение, в котором два или более элемента будут соединены вместе и, таким образом, имеют общую точку для более чем одной ветви.

Создайте бесплатную учетную запись, чтобы продолжить чтение

  • Получайте мгновенные оповещения о вакансиях бесплатно!

  • Получите ежедневную капсулу GK и текущих новостей и PDF-файлы

  • Получите 100+ бесплатных пробных тестов и викторин


Подпишитесь бесплатно Уже есть аккаунт? Войти

Следующее сообщение

KVL / KCL

Чтобы понять KCL, нам нужно поговорить о схемном узле.Это просто точка, в которой два или более компонентов электрически соединены. Любой из компонентов может протолкнуть ток в узел или извлечь его из узел. Обычно я думаю об этом токе как об электронах, хотя это может быть любая заряженная частица.

Закон KCL основан на идее, что электроны могут проходить через узел, но им не разрешается оставаться там в течение длительного периода времени.KCL — это электрический эквивалент закона отсутствия бездельничанья для электронов.

Ток указывает количество электронов, движущихся по проводу. каждую секунду (умноженное на постоянную масштабирования). Чтобы электроны не собирались в узле природа гарантирует, что сумма всех токов, входящих в узел всегда равняется сумме токов, выходящих из узла.

Существует два разных соглашения о записи токов. Один — всегда используйте положительные числа для описания силы тока, а затем используйте направление стрелки на схеме, чтобы указать, в каком направлении течет ток. Другое соглашение — разрешить токам иметь как положительные, так и отрицательные ценности.Положительное значение означает, что ток течет в направлении, указанном значком. стрелка на схеме, а отрицательный ток указывает, что ток течет в противоположном направлении. направление.

Если вы используете соглашение, согласно которому токи могут быть положительными или отрицательными числами, тогда KCL можно сформулировать так: сумма всех токов, входящих в узел, равна нулю. Если к узлу подключено четыре устройства, то схема, показывающая токи, будет выгляди так

тогда KCL говорит \ [{I_1} + {I_2} + {I_3} + {I_4} = 0 \]

Нет ничего особенного в четырех подключениях к узлу.N {{I_k}} \]

Если вам удобнее, вы можете записать KCL как сумму всех выходящих токов узел должно быть равно нулю. Но если вы допускаете в своих расчетах как положительные, так и отрицательные токи, это Важно, чтобы у вас либо все токи входили, либо все токи выходили из узла.Не смешивайте два.

Закон KCL — это приближение, которое не всегда справедливо. KCL может быть недействителен в течение коротких периодов времени.Можно поставить отрицательный электрический заряд на металлическом шаре, толкая на него электроны. Вопреки предположению В случае KCL эти электроны остаются на шаре.

Причина, по которой мы все еще используем KCL, заключается в том, что количество электронов, участвующих в статическом зарядов обычно тривиально мало — по сравнению с количеством электронов, которые текут в секунду при типичных электрических токах.

Другая проблема с KCL возникает, когда узел физически большой. Сборы не может мгновенно перемещаться из одного места в другое, проблема, которую мы увидим во многих Более подробно мы рассмотрим линии передачи и антенны.

Поставщики и ресурсы беспроводной связи RF

О мире беспроводной связи RF

Веб-сайт RF Wireless World является домом для поставщиков и ресурсов радиочастотной и беспроводной связи.На сайте представлены статьи, руководства, поставщики, терминология, исходный код (VHDL, Verilog, MATLAB, Labview), тестирование и измерения, калькуляторы, новости, книги, загрузки и многое другое.

Сайт RF Wireless World охватывает ресурсы по различным темам, таким как RF, беспроводная связь, vsat, спутник, радар, оптоволокно, микроволновая печь, wimax, wlan, zigbee, LTE, 5G NR, GSM, GPRS, GPS, WCDMA, UMTS, TDSCDMA, Bluetooth, Lightwave RF, z-wave, Интернет вещей (IoT), M2M, Ethernet и т. Д. Эти ресурсы основаны на стандартах IEEE и 3GPP.Он также имеет академический раздел, который охватывает колледжи и университеты по инженерным дисциплинам и MBA.

Статьи о системах на основе Интернета вещей

Система обнаружения падений для пожилых людей на основе Интернета вещей : В статье рассматривается архитектура системы обнаружения падений, используемой для пожилых людей. В нем упоминаются преимущества или преимущества системы обнаружения падений Интернета вещей. Читать дальше➤
Также обратитесь к другим статьям о системах на основе Интернета вещей следующим образом:
• Система очистки туалетов самолета. • Система измерения столкновений • Система отслеживания скоропортящихся продуктов и овощей • Система помощи водителю • Система умной торговли • Система мониторинга качества воды. • Система Smart Grid • Система умного освещения на базе Zigbee • Умная парковка на базе Zigbee • Система умной парковки на основе LoRaWAN


RF Статьи о беспроводной связи

В этом разделе статей представлены статьи о физическом уровне (PHY), уровне MAC, стеке протоколов и сетевой архитектуре на основе WLAN, WiMAX, zigbee, GSM, GPRS, TD-SCDMA, LTE, 5G NR, VSAT, Gigabit Ethernet на основе IEEE / 3GPP и т. Д. .стандарты. Он также охватывает статьи, относящиеся к испытаниям и измерениям, по тестированию на соответствие, используемым для испытаний устройств на соответствие RF / PHY. УКАЗАТЕЛЬ СТАТЬИ ДЛЯ ССЫЛКИ >>.


Физический уровень 5G NR : Обработка физического уровня для канала 5G NR PDSCH и канала 5G NR PUSCH рассмотрена поэтапно. Это описание физического уровня 5G соответствует спецификациям физического уровня 3GPP. Читать дальше➤


Основы повторителей и типы повторителей : В нем объясняются функции различных типов ретрансляторов, используемых в беспроводных технологиях.Читать дальше➤


Основы и типы замирания : В этой статье описываются мелкомасштабные замирания, крупномасштабные замирания, медленные, быстрые и т. Д., Используемые в беспроводной связи. Читать дальше➤


Архитектура сотового телефона 5G : В этой статье рассматривается блок-схема сотового телефона 5G с внутренними модулями 5G. Архитектура сотового телефона. Читать дальше➤


Основы помех и типы помех: В этой статье рассматриваются помехи в соседнем канале, помехи в совмещенном канале, Электромагнитные помехи, ICI, ISI, световые помехи, звуковые помехи и т. Д.Читать дальше➤


5G NR Раздел

В этом разделе рассматриваются функции 5G NR (New Radio), нумерология, диапазоны, архитектура, развертывание, стек протоколов (PHY, MAC, RLC, PDCP, RRC) и т. Д. 5G NR Краткий указатель ссылок >>
• Мини-слот 5G NR • Часть полосы пропускания 5G NR • 5G NR CORESET • Форматы DCI 5G NR • 5G NR UCI • Форматы слотов 5G NR • IE 5G NR RRC • 5G NR SSB, SS, PBCH • 5G NR PRACH • 5G NR PDCCH • 5G NR PUCCH • Эталонные сигналы 5G NR • 5G NR m-последовательность • Золотая последовательность 5G NR • 5G NR Zadoff Chu Sequence • Физический уровень 5G NR • Уровень MAC 5G NR • Уровень 5G NR RLC • Уровень 5G NR PDCP


Учебные пособия по беспроводным технологиям

В этом разделе рассматриваются учебные пособия по радиочастотам и беспроводной связи.Он охватывает учебные пособия по таким темам, как сотовая связь, WLAN (11ac, 11ad), wimax, bluetooth, zigbee, zwave, LTE, DSP, GSM, GPRS, GPS, UMTS, CDMA, UWB, RFID, радар, VSAT, спутник, WLAN, волновод, антенна, фемтосота, тестирование и измерения, IoT и т. Д. См. УКАЗАТЕЛЬ >>


Учебное пособие по 5G — В этом учебном пособии по 5G также рассматриваются следующие подтемы по технологии 5G:
Учебное пособие по основам 5G Частотные диапазоны руководство по миллиметровым волнам Волновая рама 5G мм Зондирование волнового канала 5G мм 4G против 5G Испытательное оборудование 5G Сетевая архитектура 5G Сетевые интерфейсы 5G NR канальное зондирование Типы каналов 5G FDD против TDD Разделение сети 5G NR Что такое 5G NR Режимы развертывания 5G NR Что такое 5G TF


Этот учебник GSM охватывает основы GSM, архитектуру сети, элементы сети, системные спецификации, приложения, Типы пакетов GSM, структура или иерархия кадров GSM, логические каналы, физические каналы, Физический уровень GSM или обработка речи, вход в сеть мобильного телефона GSM, установка вызова или процедура включения питания, MO-вызов, MT-вызов, VAMOS, AMR, MSK, модуляция GMSK, физический уровень, стек протоколов, основы работы с мобильным телефоном, Планирование RF, нисходящая линия связи PS и восходящая линия связи PS.
➤Подробнее.

LTE ​​Tutorial , охватывающий архитектуру системы LTE, охватывающий основы LTE EUTRAN и LTE Evolved Packet Core (EPC). Он обеспечивает связь с обзором системы LTE, радиоинтерфейсом LTE, терминологией LTE, категориями LTE UE, структурой кадра LTE, физическим уровнем LTE, Стек протоколов LTE, каналы LTE (логические, транспортные, физические), пропускная способность LTE, агрегация несущих LTE, передача голоса по LTE, расширенный LTE, Поставщики LTE и LTE vs LTE продвинутые.➤Подробнее.


RF Technology Stuff

Эта страница мира беспроводной радиосвязи описывает пошаговое проектирование преобразователя частоты RF на примере преобразователя RF UP от 70 МГц до диапазона C. для микрополосковой платы с использованием дискретных радиочастотных компонентов, а именно. Смесители, гетеродин, MMIC, синтезатор, опорный генератор OCXO, колодки аттенюатора. ➤Подробнее.
➤Проектирование и разработка радиочастотного трансивера ➤Конструкция RF-фильтра ➤Система VSAT ➤Типы и основы микрополосковой печати ➤ОсновыWaveguide


Секция испытаний и измерений

В этом разделе рассматриваются контрольно-измерительные ресурсы, испытательное и измерительное оборудование для тестирования DUT на основе Стандарты WLAN, WiMAX, Zigbee, Bluetooth, GSM, UMTS, LTE.УКАЗАТЕЛЬ испытаний и измерений >>
➤Система PXI для T&M. ➤ Генерация и анализ сигналов ➤Измерения слоя PHY ➤Тест устройства на соответствие WiMAX ➤ Тест на соответствие Zigbee ➤ Тест на соответствие LTE UE ➤Тест на соответствие TD-SCDMA


Волоконно-оптическая технология

Оптоволоконный компонент , основы, включая детектор, оптический соединитель, изолятор, циркулятор, переключатели, усилитель, фильтр, эквалайзер, мультиплексор, разъемы, демультиплексор и т. д.Эти компоненты используются в оптоволоконной связи. Оптические компоненты INDEX >>
➤Учебное пособие по оптоволоконной связи ➤APS в SDH ➤SONET основы ➤SDH Каркасная конструкция ➤SONET против SDH


Поставщики, производители радиочастотных беспроводных устройств

Сайт RF Wireless World охватывает производителей и поставщиков различных радиочастотных компонентов, систем и подсистем для ярких приложений, см. ИНДЕКС поставщиков >>.

Поставщики радиочастотных компонентов, включая радиочастотный изолятор, радиочастотный циркулятор, радиочастотный смеситель, радиочастотный усилитель, радиочастотный адаптер, радиочастотный разъем, радиочастотный модулятор, радиочастотный трансивер, PLL, VCO, синтезатор, антенну, генератор, делитель мощности, сумматор мощности, фильтр, аттенюатор, диплексор, дуплексер, микросхема резистора, микросхема конденсатора, индуктор микросхемы, ответвитель, оборудование ЭМС, программное обеспечение для проектирования радиочастот, диэлектрический материал, диод и т. д.Производители RF компонентов >>
➤Базовая станция LTE ➤RF Циркулятор ➤RF Изолятор ➤Кристаллический осциллятор


MATLAB, Labview, встроенные исходные коды

Раздел исходного кода RF Wireless World охватывает коды, связанные с языками программирования MATLAB, VHDL, VERILOG и LABVIEW. Эти коды полезны для новичков в этих языках. ИНДЕКС ИСХОДНОГО КОДА >>
➤3-8 декодер кода VHDL ➤Код MATLAB для дескремблера ➤32-битный код ALU Verilog ➤T, D, JK, SR триггеры labview коды


* Общая информация о здоровье населения *

Выполните эти пять простых действий, чтобы остановить коронавирус (COVID-19).
СДЕЛАЙТЕ ПЯТЬ
1. РУКИ: часто мойте их
2. КОЛЕНО: Откашляйтесь
3. ЛИЦО: не трогай его
4. НОГИ: держитесь на расстоянии более 3 футов (1 м) друг от друга
5. ЧУВСТВОВАТЬ: Болен? Оставайся дома

Используйте технологию отслеживания контактов >>, соблюдайте >> рекомендации по социальному дистанцированию и установить систему видеонаблюдения >> чтобы спасти сотни жизней. Использование концепции телемедицины стало очень популярным в таким странам, как США и Китай, остановить распространение COVID-19, поскольку это заразное заболевание.


RF Беспроводные калькуляторы и преобразователи

Раздел «Калькуляторы и преобразователи» охватывает ВЧ-калькуляторы, беспроводные калькуляторы, а также преобразователи единиц. Сюда входят такие беспроводные технологии, как GSM, UMTS, LTE, 5G NR и т. Д. СПРАВОЧНЫЕ КАЛЬКУЛЯТОРЫ Указатель >>.
➤ Калькулятор пропускной способности 5G NR ➤5G NR ARFCN против преобразования частоты ➤Калькулятор скорости передачи данных LoRa ➤LTE EARFCN для преобразования частоты ➤Калькулятор антенн Яги ➤ Калькулятор времени выборки 5G NR


IoT-Интернет вещей Беспроводные технологии

Раздел IoT охватывает беспроводные технологии Интернета вещей, такие как WLAN, WiMAX, Zigbee, Z-wave, UMTS, LTE, GSM, GPRS, THREAD, EnOcean, LoRa, SIGFOX, WHDI, Ethernet, 6LoWPAN, RF4CE, Bluetooth, Bluetooth Low Power (BLE), NFC, RFID, INSTEON, X10, KNX, ANT +, Wavenis, Dash7, HomePlug и другие.Он также охватывает датчики Интернета вещей, компоненты Интернета вещей и компании Интернета вещей.
См. Главную страницу IoT >> и следующие ссылки.
➤ НИТЬ ➤EnOcean ➤Учебник по LoRa ➤Учебник по SIGFOX ➤WHDI ➤6LoWPAN ➤Zigbee RF4CE ➤NFC ➤Lonworks ➤CEBus ➤UPB



СВЯЗАННЫЕ ЗАПИСИ


RF Wireless Tutorials



Различные типы датчиков


Поделиться страницей

Перевести страницу

Как решить сложные схемы с помощью Кирхгофа…

Мы рассмотрели Закон Кирхгофа (KCL) в предыдущем учебном пособии, а Закон напряжения Кирхгофа (KVL) очень похож, но сосредоточен на напряжении в цепи, а не на токе.Закон Кирхгофа о напряжении гласит, что сумма напряжений в замкнутом контуре равна нулю. Другими словами, если вы посмотрите на любую петлю, которая идет полностью по всему периметру, любое увеличение напряжения по всей петле будет компенсировано равным уменьшением напряжения. Визуально это можно увидеть на изображении ниже.

Используя эту концепцию, так же, как мы можем использовать узловой анализ с KCL, мы можем использовать анализ сетки из-за KVL. Хотя сетка — это, по сути, любой цикл в цепи, для анализа сетки нам нужно будет определить сетки, которые не охватывают никакие другие сетки.

Вы можете видеть, что если мы сделаем цикл вокруг «внешней части» всей схемы, технически это будет сетка, потому что цикл может быть завершен. Однако для анализа нам нужно разбить его на три разные сетки. Итак, давайте рассмотрим шаги, как решить схему с помощью анализа сетки, прежде чем переходить к нескольким примерам.

Есть 5 шагов, которые мы рекомендуем, и, как мы делали с шагами KCL / узлового анализа, два шага — успокоиться и отступить, убедившись, что все интуитивно понятно.

  1. Не торопитесь, подышите и оцените проблему. Запишите, какая информация вам была предоставлена, и какие у вас есть интуитивные идеи.
  2. Назначьте токи сетки всем сеткам. На сетку должен быть назначен один ток. Вам нужно выбрать, в каком направлении течет ваш ток — это наполовину произвольно, потому что, пока вы правильно рассчитываете, это не имеет значения. Но в большинстве случаев люди принимают направление тока по часовой стрелке.
  3. Примените KVL к каждой из ячеек, используя закон Ома, чтобы показать напряжения через ток.
  4. Решите одновременные уравнения (как мы это делали с KCL), чтобы найти фактические значения.
  5. Проверка работоспособности. Найдите минутку, чтобы проанализировать, что вы сделали, и посмотрите, имеют ли цифры смысл и внутренне согласованы.

Сейчас мы рассмотрим несколько примеров, и, честно говоря, после этих примеров единственными реальными дополнениями и изменениями будут сложности, которые усложняют математические вычисления. Концептуально задачи не должны становиться намного сложнее, но математика может стать значительно сложнее.Пожалуйста, не теряйтесь в математике. Если числа начинают терять свои числа, не забудьте выпустить воздух и вспомнить, что вы делаете и что пытаетесь сделать.


Пример 1

Простой пример — 1 сетка.

Начнем здесь! Это простая схема, настолько простая, что мы можем решить эту проблему с помощью уже известных инструментов. Но я хочу начать с простого, чтобы мы могли сосредоточиться на концепциях и шагах. Итак, давайте сделаем это.

Шаг 1. Подведем итоги схемы.Очевидно, у него только одна петля, и у нас есть источник напряжения и два резистора. Нам даны значения источника напряжения и обоих резисторов, поэтому все, что нам нужно, это узнать ток в контуре и падение напряжения на резисторах. И как только мы находим одно, мы можем быстро использовать закон Ома, чтобы получить другое. Это будет легко.

Шаг 2: Мы уже заметили на шаге 1, что будет только одна сетка, поэтому давайте нарисуем нашу сетку текущую, зададим ей направление и дадим имя.Пойдем по часовой стрелке и назовем его 1 . Я обычно неаккуратен и не различаю i 1 и I 1 , но в этом случае мы будем использовать строчную букву «i». Это будет важно в следующих примерах. И мы знаем, что, поскольку у нас есть одна сетка, будет только одно уравнение.

Шаг 3. Давайте создадим наши уравнения на основе KVL. Это первый шаг, требующий математических вычислений. Итак, с помощью KVL давайте разберемся с нашим уравнением.

Есть два взгляда на это, которые могут вызвать неописуемую путаницу.Я объясню различия, и, если вы будете последовательны в каждом уравнении (даже не обязательно в каждой проблеме, но черт возьми, зачем вам без надобности запутывать себя?), Тогда все будет хорошо.

В первом варианте, когда мы обходим контур, мы видим, что мы увеличиваем на 5 В на источнике напряжения, а затем падаем напряжение на R 1 и R 2 , давая нам наши положительные 5 вольт, а затем наши два негативы. Для меня это более интуитивно понятно, потому что вы повышаете напряжение на источнике напряжения так, как мы определили поток тока, и вы понижаете напряжение на резисторах по мере прохождения через них тока.Однако очень часто люди изучают его вторым способом.

Во втором варианте вы просто используете знак напряжения на той стороне вашей ветви, в которую входит ток. В случае источника напряжения, поскольку мы движемся по часовой стрелке, ток сначала видит отрицательный знак, так что это минус. По мере того, как напряжение на резисторах падает с положительного на отрицательный, ток сначала видит положительный знак на резисторах, поэтому вы добавляете их. Если это для вас более интуитивно понятно — воспользуйтесь! Ни один из этих вариантов не является неправильным, вы видите, что вы получаете те же уравнения (просто умножьте обе стороны на -1), но убедитесь, что вы согласны с каждым уравнением.Пожалуйста.

Шаг 4: Поскольку неизвестных нет, мы можем просто подставить значения для R 1 и R 2 и узнать, что такое i 1 .

А теперь мы можем найти напряжения на R 1 и R 2 .

Шаг 5: Проверка работоспособности! Обратите внимание, что V 1 + V 2 в основном равняется 5 В (ошибки округления!), Что означает, что напряжение, которое падает на двух резисторах, совпадает с увеличением напряжения от источника напряжения.

Давайте немного усложним ситуацию.


Пример 2

Шаг 1: Что мы здесь получили? Похоже, у нас есть две сетки, которые имеют общий резистор посередине, R 3 . Опять же, у нас есть все значения источников напряжения и резисторов, поэтому мы должны иметь возможность получить фактические значения для тока и напряжения через эти резисторы. Даже без каких-либо значений мы могли бы провести анализ и показать взаимосвязи, но мне немного приятнее прийти к числовому ответу.Нам действительно нужно знать, как обращаться с R 3 , но мы позаботимся об этом на шаге 3.

Шаг 2: Давайте определим сетки. Мы заставим обе токовые петли течь по часовой стрелке и назовем левую i 1 , а правую — 2 . Обратите внимание, что это все еще строчные буквы. И на этот раз это имеет значение, потому что у нас также есть ток через резистор R 1 , который равен I 1 (обратите внимание на букву «I» с большой буквы), ток через резистор R 2 , то есть I 2 , а затем через R 3 , то есть I 3 .Использование заглавных букв — это то, как различать токи сетки (i 1 и i 2 ) и токи ответвления (I 1 , I 2 и I 3 ).

Шаг 3: Создайте уравнения для сеток. Это будет довольно просто, но нам нужно знать, что делать с напряжением на R 3 . Давайте на самом деле составим уравнение для i 1 , а затем немного поговорим о нем.

Итак, глядя на это уравнение, вы, вероятно, задаетесь вопросом, почему в нашем уравнении для тока сетки i 1 содержится i 2 .Помните, что каждая секция относится к напряжению. Мы увеличиваем на 10В, что несложно. Мы понижаем напряжение на R 1 , что равно i 1 * R 2 , все еще довольно просто. Но падение напряжения на R 3 — это величина тока, текущего вниз, как i 1 , минус величина тока, протекающего вверх, как i 2 , умноженная на R 3 .

В нашем направлении по часовой стрелке мы заявили, что i 2 течет с вверх по через 3 рэндов.Очевидно, что на самом деле ток течет только в одну сторону, но мы не знаем, в какую сторону прямо сейчас, и математически мы сказали, что есть оба тока, протекающие через R 3 , как 1 и текущие через R 3 как i 2 . Уловка здесь в том, что если бы мы определили i 2 в противоположном (против часовой стрелки) направлении, нам пришлось бы добавить к току i 2 к i 1 , чтобы вычислить падение напряжения на R 3 .

Итак, сделайте паузу, остановитесь на секунду, убедитесь, что вы понимаете, почему мы создали уравнение, которое мы сделали для тока первой сетки. Затем посмотрите, что вы получите для второго тока сетки, прежде чем проверять, что мы получим. Однако вам придется контролировать свои глаза, потому что ответ находится прямо под этим текстом.

Это то, что у вас есть? Помните, что с нашим определением, что ток течет по часовой стрелке, напряжение падает, когда мы идем от земли через R 3 , и все еще падает, когда мы идем через R 2 , прежде чем подойти к источнику напряжения, который, поскольку мы Определив это направление по часовой стрелке, мы получаем отрицательные 5 вольт.Именно здесь невероятно важно понимать, что происходит интуитивно — если вы слишком увязнете в математике, не зная, что происходит, вы будете составлять и решать неправильные уравнения! Поверьте мне — я говорю из очень болезненного опыта.

Итак, теперь у нас есть два уравнения и два неизвестных. Мы можем либо решить эту проблему с помощью подстановки, либо подготовившись к некоторой линейной алгебре. Сделаем замену.

Введите значения резисторов.

Упростим первое уравнение.

Перед заменой i 1 немного упростите второе уравнение.

Пример 3 (Суперсетки)

С KCL, если бы у нас был источник напряжения, который не был напрямую подключен к опорной земле, мы бы создали суперузел, а затем, как часть процесса, нам нужно было бы сделать немного КВЛ, чтобы закончить анализ. С KVL, если у нас есть текущий источник, который используется двумя сетками, мы должны относиться к нему аналогичным образом. Мы избавляемся от текущего источника и всего, что связано с ним последовательно.Затем мы рассматриваем оставшуюся часть как одну большую суперсетку.

После создания этой сетки мы создаем уравнение для ее описания. В этом случае мы получаем:

Теперь у нас есть уравнение для супер-сетки, но у нас есть два неизвестных и только одно уравнение. Итак, давайте снова подключим источник тока к любым элементам, которые были с ним последовательно, и проведем KCL в узле, где они подключаются к большей цепи. Как только это будет сделано, мы используем KCL в этом узле, чтобы создать второе уравнение.

Теперь у нас есть два уравнения и два неизвестных! Давайте представим это в формате, необходимом для выполнения некоторой линейной алгебры, и посмотрим, что у нас получится.

Итак, наши два уравнения:

Которые мы помещаем в программу решения линейных уравнений, чтобы получить:

Поскольку я склонен делать математические ошибки, я предпочитаю метод линейных уравнений, поскольку он обычно быстрее и менее вероятно, что я ошибаюсь. вверх это. В случае супер-сетки это не обычная проблема, поскольку, если вы не имеете дело с транзисторами или схемой уровня CMOS, источники тока не очень типичны. Однако это хороший инструмент на случай, если он возникнет, и он поможет нам лучше понять взаимосвязь между физическими схемами и математическими представлениями.


Резюме

Это наш краткий обзор закона напряжения Кирхгофа и того, как он приводит к анализу сетки. Вы заметите, что иногда мы использовали анализ сетки и KVL как синонимы. Хотя технически это не то же самое, очень часто можно услышать, как их используют таким образом. В зависимости от того, где вы находитесь и с кем учились, вы можете обнаружить некоторые другие различия в подходах, соглашениях об именах и даже в предположениях относительно направления. Однако, несмотря на эти внешние различия, весь анализ сетки сводится к нахождению напряжения на различных элементах сетки.Если вы последовательны и хорошо понимаете, что делаете, вы сможете получить ответ, который ищете.

Закон напряжения Кирхгофа (KVL)

Закон напряжения Кирхгофа, утверждает, что сумма всех замкнутых контуров в цепи должна быть равна нулю.

ЦЕПЬ ЗАКОНА НАПРЯЖЕНИЯ Кирхгофа (KVL):

Принцип, известный как Закон Кирхгофа в 1847 году немецким физиком Густавом Р. Кирхгофом, может быть сформулирован следующим образом:

« сумма всех напряжений в контуре должна равняться нулю.

Это связано с тем, что контур контура может быть замкнутым проводящим путем, таким образом, энергия не теряется.

Математически ΣV = 0.

Пример закона Кирхгофа, №1:

Три резистора номиналом: 20 Ом, 30 Ом и 40 Ом соответственно подключены последовательно через 24-вольтовый аккумулятор. Рассчитайте: а) полное сопротивление, б) ток цепи, в) ток через каждый резистор, г) падение напряжения на каждом резисторе, д) убедитесь, что закон Кирхгофа для напряжения KVL верен.

Решение:

a) Общее сопротивление (R
T ):

R T = R 1 + R 2 + R 3 = 20 Ом + 30 Ом + 40 Ом = 90 Ом

Тогда полное сопротивление цепи R T равно 90 Ом.

б) Ток цепи (I):

Таким образом, общий ток цепи равен 0,26 ампера.

c) Ток через каждый резистор:

Резисторы соединены последовательно, все они являются частью одного контура и, следовательно, каждый испытывает одинаковое количество тока. Таким образом:

I R1 = I R2 = I R3 = I СЕРИЯ = 0,26 ампера

г) Падение напряжения на каждом резисторе:

В R1 = I х 1 = 0.267 x 20 = 5,34 В

В R2 = I x R 2 = 0,267 x 30 = 8,01 В

В R3 = I x R 3 = 0,267 x 40 = 10,68 В

e) Проверьте закон Кирхгофа о напряжении:

Таким образом, (kvl) остается верным, поскольку отдельные падения напряжения вокруг замкнутого контура складываются в общее.

F) Цепь цепи:

Как это:

Нравится Загрузка.

Добавить комментарий

Ваш адрес email не будет опубликован. Обязательные поля помечены *